Uploaded by turboroom

ТЕХНИЧЕСКАЯ ГАЗОДИНАМИКА(new111)

advertisement
ГОСУДАРСТВЕННЫЙ КОМИТЕТ РФ ПО РЫБОЛОВСТВУ
Федеральное государственное образовательное учреждение
высшего профессионального образования
Дальневосточный государственный технический
рыбохозяйственный университет
ФГОУ ВПО «ДАЛЬРЫБВТУЗ»
Одобрено на заседании кафедры "Судовые энергетические установки"
УДК 629.12.06
ББК 39.42-04
А 861
Техническая газовая динамика: законы сохранения.
Учебное пособие /сост. Ю.Я. Фершалов, М.Ю. Фершалов, А.Ю. Фершалов. –
Владивосток: Изд-во Дальневосточного государственного рыбохозяйственного университета
(Дальрыбвтуз), 2008. – 96 с.
Ю.Я. Фершалов, М.Ю. Фершалов, А.Ю. Фершалов
ТЕХНИЧЕСКАЯ ГАЗОДИНАМИКА:
Учебное пособие
По практическим занятиям и выполнению
контрольных работ
для студентов и курсантов специальности 180403
всех форм обучения
Учебное пособие предназначено для организации самостоятельной работы студентов специальности 240500 "Эксплуатация судовых энергетических установок"
дневного и заочного форм обучения.
Учебное пособие содержит теоретические положения и примеры
решения задач с использованием законов сохранения применительно к задачам технической газовой динамики. Приведены условия задач, предназначенные для выполнения самостоятельной работы, с целью практического
усвоения курса "Техническая газовая динамика".
Редактор
Техн. редактор
Подписано в печать
. Формат 60x84/16
Усл. печ. л.
Уч.- изд. л.
Тираж 100 экз. Заказ
.
__________________________________________________________
___________
Издательство Дальрыбвтуз
Типография издательства Дальрыбвтуз
Владивосток
2008
© Ю.Я. Фершалов,
М.Ю. Фершалов,
А.Ю. Фершалов 2008
© Дальрыбвтуз, Изд-во Дальрыбвтуз,2008
Условные обозначения
Е – энергия;
τ – время, касательное напряжение;
с – скорость движения;
L – работа.
Р – давление;
р – вес;
Индексы
Т – абсолютная температура;
ρ – плотность;
m – масса;
н – табличные параметры;
V – объем;
в – табличные значения максимально возможных параметров пара,
V – удельный объем;
до превращения его в воду;
μ – молекулярная масса;
0; 1; 2 – порядковые номера;
R – газовая постоянная (равна отношению универсальной газовой
Δ – приращение;
постоянной на молекулярную массу газа - 8314,41/µГ);
n – потенциальная;
φ – относительная влажность газа;
внут. – внутренняя;
γ – удельный вес;
к – кинетическая;
g – ускорение свободного падения;
вн. – внешняя;
F – сила;
турб – турбина;
η – динамическая вязкость;
компр – компрессор;
ν – кинематическая вязкость;
* – параметры торможения;
β – коэффициент сжимаемости;
u – окружная составляющая скорости.
М – число Маха;
h – энтальпия;
ср – удельная изобарная теплоемкость газа (теплоемкость при
постоянном давлении);
ср – удельная изохорная теплоемкость газа (теплоемкость при
постоянном объеме);
k – коэффициент изоэнтропы;
S – энтропия;
Q –теплота;
G – расход газа;
W – тепловая энергия;
2
Введение
Часть этого раздела, относящаяся к жидким и газообразным средам, называют механикой жидкости и газа.
Главная задача вуза научить слушателей самостоятельно решать
В отличие от газа, в жидкости расстояния между молекулами очень
встающие перед ними технические проблемы. Специальные дисциплины,
малы. Это приводит к возникновению значительных молекулярных сил
изучаемые на старших курсах, по существу состоят из систематически изла-
сцепления, что делает жидкость мало сжимаемой. Поэтому при описании
гаемых решений ряда важнейших для изучаемого предмета задач. Основой
процессов происходящих в современных технических устройствах жидкость
для этого служат законы фундаментальных наук (к коим относится предмет
можно рассматривать как несжимаемое вещество. В связи с этим из раздела
"Техническая газовая динамика"), преподавание которых ведется на млад-
механики жидкостей и газов выделился раздел "Газовая динамика", в кото-
ших курсах.
ром изучается движение сжимаемых сред и их взаимодействие с твердыми
телами.
Слушать лекции и изучать предмет по учебникам не достаточно для
полного развития творческих способностей будущего инженера.
Движение газа подчиняется общим законам механики. Среди них
Для этой цели служат различные формы самостоятельной работы
особенно важное и наиболее общее значение имеют законы сохранения
студентов. Среди них одна из важнейших – постоянная тренировка в реше-
следующих величин: энергии, количества движения (импульса), момента
нии задач, систематически подобранных в специальных пособиях. Без них
количества движения (момента импульса). Законы сохранения используют
изучение любого изучаемого курса не может быть полноценным.
в любом разделе физики. Особенность применения их в газовой динамике
Среди освоенных судовыми механиками предметов видное место
заключается в том, что эти законы надо описать в форме, пригодной для
принадлежит "Технической газовой динамике".
изучения движения сплошной деформируемой среды.
При изучении форм движения и взаимодействия материальных тел
В данной работе рассматриваются явления, происходящие при
используется такое абстрактное понятие, как материальная точка или си-
движении газа в технических устройствах, где газ движется со скоростями
стема материальных точек. Материальная система может рассматривать-
значительно меньшими, чем скорость света, благодаря чему можно прене-
ся как дискретная система (состоящая из отдельных материальных точек),
бречь релятивистскими эффектами. Поэтому к вышеперечисленным зако-
или как сплошная среда, представляющая собой непрерывное распределе-
нам сохранения добавлен закон сохранения массы.
ние массы вещества, а также непрерывность физических характеристик его
При конструировании и эксплуатации многих типов современных
состояния и движения в пространстве.
машин и конструкций, к каким относятся судовые механизмы и судно в це-
Сплошная среда рассматривается в двух вариантах:
лом, необходимы глубокие знания в этой области. Знание газовой динамики
• - неизменяемая среда – представляет собой абсолютно твердое
одинаково важно как для конструкторов и инженеров, так и для плавсостава.
тело;
Конструктор, пользуясь законами и методами газовой динамики,
• - изменяемая сплошная среда, которая представляет собой пла-
имеет возможность выбрать наилучшие формы и размеры проектируемого
стичные, жидкие и газообразные объекты.
механизма и рассчитать его характеристики.
Раздел теоретической механики, занимающийся изучением движения изменяемых сплошных сред, называется механикой сплошной среды.
3
1 ОСНОВНЫЕ ГИПОТЕЗЫ МЕХАНИКИ
Инженер, руководящий технической эксплуатацией и обслуживани-
СПЛОШНОЙ СРЕДЫ
ем механизмов, обязан отчетливо понимать зависимость их свойств от
условий эксплуатации, ремонта и обслуживания.
Для описания поведения газа необходимы полные и непротиворе-
Судовой механик должен овладеть этой наукой для сознательного
чивые модели движения газообразных объектов, основанные на методах
обслуживания устройств, добиться полного использования их возможностей.
теоретической механики и некоторых дополнительных гипотезах. Согласо-
Пособие рассчитано на слушателей дневной и заочной форм обу-
ванная система таких моделей носит название механики сплошной среды.
чения, обучающихся по направлению подготовки дипломированных специа-
Одной из важнейших особенностей газовой динамики является то,
листов специальности 658000 "Эксплуатация водного транспорта и транс-
что в основу ее положена так называемая модель сплошной среды. Как из-
портного оборудования" и 240500 "Эксплуатация судовых энергетических
вестно, для описания среды, состоящей из большого числа молекул в срав-
установок", а также и других специальностей.
нительно малом объеме (газы) в физике широко используются два пути:
Эта работа предназначена для самостоятельного изучения студен-
феноменологический и статистический. Феноменологический путь изучения
тами данного предмета, чтобы понимать сущность возникающих при эксплу-
основывается на простейших допущениях. Оставляя в стороне вопрос о
атации механизмов и устройств явлений, и иметь возможность сознательно
строении вещества, он наделяет его такими свойствами, которые наилуч-
выполнять предписания инструкции по их техническому обслуживанию.
шим образом устанавливают соответствие между наблюдаемыми явлениями и их описанием.
При таком подходе газы рассматриваются как непрерывная среда,
способная делиться до бесконечности. Другими словами, газ представляется состоящими из достаточно малых частиц непрерывным образом заполняющих пространство. Эта среда обладает свойством инерции и наделена
различными физическими свойствами. Согласно такой модели все параметры газа (плотность, вязкость и др.) изменяются непрерывно от точки к точке.
Это позволяет при анализе движения газа применять математический аппарат дифференциального и интегрального исчислений хорошо разработанный для непрерывных функций.
Понятие о частицах жидкости, которым широко оперирует газовая
динамика, неразрывно связано с понятием о физически бесконечно малом
объеме. Это объем, размеры которого пренебрежимо малы по сравнению с
характерными размерами объекта, но он содержит в себе настолько много
молекул, что его средние характеристики (например, плотность) становятся
устойчивыми по отношению к изменению объема. Поэтому, например, фраза «объем стягивается в точку» означает, что он стремится не к нулю, а к
4
физически бесконечно малому объему. Следует твердо усвоить, что все за-
Совершенный газ – это газ, в котором отсутствуют силы межмоле-
коны механики жидкости справедливы до тех пор, пока справедлива
кулярного взаимодействия. В термодинамике такой газ называют идеальным
модель сплошной среды. Количественно это можно оценить по величине
газом.
числа Кнудсена, представляющего отношение длины свободного пробега
Совершенный газ имеет постоянные удельные теплоемкости при
молекул l к характерному размеру течения L
постоянном объеме и давлении (соответственно) и для него справедливы
следующие законы:
l
Kn 
L
• - закон Бойля-Мариотта: при неизменных температуре и массе
произведение численных значений давления и объема газа постоянно
Принято считать, что законы механики жидкости справедливы при
Kn<0,01.
Все газы состоят из множества отдельных частиц, взаимодейству-
P  V  const ;
ющих сложным образом в электромагнитном, гравитационном и других, пока
• - закон Гей-Люссака: при постоянном давлении объем данной мас-
неизвестных полях. Поэтому изучение газа как совокупности частиц требует
сы газа прямо пропорционален его абсолютной температуре
введения дополнительных гипотез об их свойствах и взаимодействии. Кроме
того, для решения уравнений динамики необходимо знать начальные усло-
V  V0
вия, т.е. координаты и скорости всех частиц, что принципиально невозможно.
В качестве основного свойства принятой модели газовой среды,
T
;
T0
• - закон Шарля: при постоянном объеме давление данной массы
лежащего в основе кинематики газа, принята ее сплошность (непрерыв-
газа прямо пропорционально его абсолютной температуре
ность распределения массы и физико-механических характеристик среды).
Для динамики газа существенно второе основное свойство газооб-
P  P0
разной среды – ее текучесть. Текучесть выражается в том, что для газов
касательные напряжения (внутреннее трение) присутствуют только при
наличии относительного движения сдвига между слоями газа. При относи-
T
;
T0
• - закон Авогадро: при одинаковых давлениях и одинаковых темпе-
тельном покое среды внутреннее трение отсутствует. В этом заключается
ратурах в равных объемах различных совершенных газов содержится оди-
отличие газообразной среды от упругой, в которой касательные напряжения
наковое число молекул;
обусловлены наличием деформаций и отличны от нуля при относительном
покое упругой среды.
• - уравнение Менделеева-Клайперона (уравнение состояния со-
Для исследования движения газа введено понятие совершенного
вершенного газа)
газа.
P    R T .
5
2. Классификация сил действующих в газе
При рассмотрении газодинамических процессов в технических конструкциях принято две простейших модели совершенного газа:
При всяком изучении физических явлений необходимо выявление
• - в идеальном газе отсутствуют внутреннее трение и теплопро-
основных факторов, определяющих поведение объекта интересующего ис-
водность;
следователя. Обоснованная классификация этих факторов позволяет упро-
• -в вязком газе присутствие трения пропорционально скорости, а
стить понимание происходящих процессов и, не взирая на некоторые допу-
наличие или отсутствие теплопроводности определяется принятым допуще-
щения, в итоге получить решение с приемлемой погрешностью.
нием.
Движение материальной сплошной среды происходит под воздей-
При изучении поведения объектов существуют различные подходы.
ствием сил. Понятие приложения силы в газовой динамике по сравнению с
Механика сплошной среды использует феноменологический под-
механикой материальной точки и твердой неизменяемой поверхности
ход, основанный на ниже приведенных эмпирических (основанных на
усложняется.
наблюдениях) гипотезах, подтвержденных экспериментальными исследова-
Как и в механике твердого тела, в гидромеханике силы классифици-
ниями.
руются по разным признакам: внутренние и внешние, сосредоточенные и
• Гипотеза сплошности, предложенная Бернулли, представляет газ
распределенные.
как непрерывную среду, заполняющую некоторый объем. Она необходима
Внутренними силами называют силы, вызванные объектами, при-
для применения математического аппарата дифференциального и инте-
надлежащими системе, движение которой рассматривается.
грального исчисления.
Внешними силами называют силы, вызванные внешними по отно-
• Гипотезу непрерывности метрического пространства, тесно свя-
шению к рассматриваемой системе объектами.
занную с предыдущей, вводят для определения координат и расстояний.
Понятие внешних и внутренних сил относительно. Так, например,
• Следующая гипотеза предполагает возможность введения единой
если рассматривать движение воздуха в атмосфере совместно с Землей, то
для всех точек пространства системы координат. Вспомним, что в де-
сила тяжести воздуха – внутренняя сила. Если же рассмотреть движение
картовой системе координат каждая точка пространства имеет свои дей-
только воздуха, то сила тяжести внешняя сила. То же самое относится и к
ствительные координаты. Эта гипотеза позволяет применять аппарат ана-
движению объекта в электромагнитных полях.
литической геометрии.
В механике твердого тела в основном имеют дело с сосредоточен-
• В механике сплошной среды постулируется абсолютность време-
ными или концентрированными силами, то есть с силами, действующими в
ни для всех систем отсчета, т.е. не учитываются эффекты теории отно-
точке.
сительности.
Особенность сил действующих в газовой среде заключается в том,
Эти гипотезы естественны с точки зрения человеческого опыта и
что в газе из-за большой подвижности частиц и слабой связи между ними,
вполне оправданы при исследовании явлений, происходящих в не слишком
могут действовать только непрерывно распределенные силы, не вызываю-
больших и не слишком малых объемах с небольшими скоростями (то есть в
щие деформации газообразного объекта. При этом они должны быть внеш-
макромире). Исходя из них, строятся все последующие положения и выводы
ними по отношению к объекту. Перевод внутренних сил в категорию внеш-
теории прикладной газовой динамики.
6
них производится методом сечений, либо методом «замораживания», суть
случае, касательные составляющие поверхностных сил определяют силы
которых сводится к тому, что в среде выделяется («замораживается») за-
трения.
мкнутый объем, внешняя среда мысленно отбрасывается и ее действие за-
Необходимо отметить, что такое разграничение физически не со-
меняется действием распределенных сил.
всем оправдано и поэтому условно. В действительности силы могут быть
Важнейшей особенностью газовой динамики как науки является то,
приложены только к частицам среды, у которых всегда присутствуют силы
что в ней, помимо приведенной выше классификации, силы разделяются на
сцепления между собой, а эти силы по своей природе объемные. Однако, в
объемные (массовые) силы и поверхностные силы.
газовой динамике не стоит задача специального рассмотрения сил воздей-
Под объемными силами понимают такие силы, которые приложены
ствия на отдельные частицы газа, поэтому такая классификация оправдана
ко всем частицам объема и пропорциональны массе этого объема вне зави-
и удобна.
симости от действительного его ускорения. Они пропорциональны массе
Кроме того, при рассмотрении взаимодействие газа с твердой по-
выделенного объема, или при постоянной плотности среды пропорциональ-
верхностью, частицы, на которых сосредоточено действие сил, расположе-
ны объему. Их важнейшей особенностью является то, что они действуют на
ны в настолько тонком слое, что можно без большой погрешности сводить
все частицы газа. В общем случае это силы, подчиняются второму закону
их к некоторой материальной поверхности и считать, что силы действуют на
Ньютона
элемент этой поверхности.
К объемным силам относят – силу веса, силу инерции, силу электромагнитного или электрического поля и т.п.
Поверхностные силы – действуют на элементы поверхности выделенного объема, они могут быть распределены на поверхностях раздела в
газе. В отличие от массовых сил, поверхностные силы действуют лишь на
частицы, находящиеся на поверхности объема газа.
Обычно поверхностные силы складываются из поверхностных сил,
направленных по нормали к выделенной площадке, и поверхностных сил,
направленных по касательной к этой площадке.
К поверхностным силам относят силы, действующие со стороны потока на поверхность погруженного в него тела или реакции тела на поток
(силы давления), а также силы внутреннего трения (вязкости) в среде.
При неподвижном газе поверхностные силы направлены по нормали к элементу поверхности выделенного объема (например, стенки баллона,
в котором газ находится под повышенным давлением). В движущемся газе
имеют место и нормальные и касательные составляющие поверхностных
сил (например, газ под повышенным давлением течет в трубе). В данном
7
3. Параметры состояния газа
4. Физический смысл некоторых величин
Если не учитывать ионизированный газ, именуемый плазмой, то
4.1. Давление
вещество обычно пребывает в одном из трех основных состояний:
Вообще давление это физическая величина, характеризующая ин-
• - газ;
• - жидкость;
тенсивность нормальных (перпендикулярных к поверхности) сил с которыми
• - твердое тело.
один объект действует на другой.
Очевидно, что одно и то же вещество при разных условиях может
Давление газа в дальнейшем будем обозначать - Р, Ïà
находиться в различных состояниях. При заданных неизменных условиях
рассматриваемое вещество будет находиться в одном и том же состоянии
   H2  .
ì

Для газа вводится понятие давления в каждой точке среды. В лю-
(например, при атмосферном давлении и температуре 500°С вода будет
бой точке покоящегося газа давление по всем направлениям одинаково.
существовать только в виде пара).
Это справедливо и для движущегося газа, если считать его идеаль-
Для определения конкретных физических условий, при которых
ным (лишенным трения). В вязком газе под "давлением в данной точке" по-
рассматривается вещество, и тем самым однозначно определяется его со-
нимают среднее давление по трем осям (X,Y,Z).
стояние, вводят характеристики состояния вещества – параметры состоя-
Давление в газовой среде связано с передачей импульса при
ния.
столкновении находящихся в движении молекул газа друг с другом или с поПод параметрами состояния понимают физические величины, ха-
верхностью.
рактеризующие состояние системы. К ним относят – температуру, давление,
Давление рассматривается в двух вариантах: абсолютное и избы-
удельный объем и т.п.
точное.
Различают экстенсивные и интенсивные параметры.
Абсолютное давление газа определяет состояние вещества и
Экстенсивные параметры – параметры пропорциональные массе
представляет собой силу, действующую по нормали к поверхности тела и
системы. К ним относят – объем, внутреннюю энергию, энтропию, энтальпию
отнесенную к единице площади этой поверхности.
и т.п.
Избыточное давление (разность между абсолютным давлением и
Интенсивные параметры – параметры, не зависящие от количества
барометрическим) не определяет состояние вещества и, следовательно, не
вещества (массы) в системе. К таким параметрам относят абсолютное дав-
является параметром состояния, поскольку зависит также от состояния
ление, температуру и другие. Если речь идет о потоке газа, к ним добавляют
окружающей среды (н а п р и м е р – давление окружающего воздуха).
скорость и ускорение.
Кроме понятий абсолютного и избыточного давления, существует
Удельные параметры (параметры, отнесенные к единице массы
понятие полного давления в потоке, которое складывается из статическо-
вещества) приобретают смысл интенсивных свойств.
го и динамического давления.
Статическое давление обусловлено потенциальной энергией газа
находящегося под давлением.
8
Динамическое давление (напор) обусловлено кинетической энергией движущегося газа.
При увеличении скорости потока динамическая составляющая ско-
4.2. Температура
рости потока растет, а статическая составляющая скорости потока падает.
Температура это физическая величина, которая характеризует со-
При экспериментальных исследованиях давление измеряют манометрами, барометрами, вакуумметрами, мановакууметрами.
стояние термодинамического равновесия макроскопической системы, явля-
Статическое давление измеряется манометром, измерительный
ется одним из важнейших параметров состояния газа. Она отражает тепло-
элемент которого установлен перпендикулярно направлению потока (рис.
вое состояние газа (степень того, как один объект нагрет относительно дру-
4.1).
гого) и пропорциональна средней кинетической энергии частиц вещества.
В общем случае температура определяется как производная от
энергии тела в целом по его энтропии (об энтропии речь пойдет ниже). А так
как кинетическая энергия всегда положительна, то и температура тоже все-
Рст
Измерительный
элемент
гда положительна. В связи с этим ее называют абсолютной температурой.
Она является мерой средней кинетической энергии поступательного движения молекул идеального газа (это справедливо в области не слишком низких
Рис. 4.1 Определение статического давления
температур (Т0>>3°K), чтобы квантовые свойства систем частиц не вызывали отклонение свойств идеального газа от свойств обычных газов).
Полное давление измеряется манометрами, измерительный эле-
За единицу абсолютной температуры в Международной системе
 
мент которых установлен параллельно и на встречу направлению потока
единиц (СИ) принят градус Кельвина - K .
(трубка Пито) (рис. 4.2).
Если изолированная система не находится в равновесии, то с течением времени переход энергии (теплопередача) от более нагретых частей
системы к менее нагретым приводит к выравниванию температуры во всей
системе.
Действительно, тепло может самопроизвольно переходить от однополное
го объекта к другому только от более нагретых объектов к менее нагретым.
Р
То есть от газа с большей температурой к газу или телу с меньшей температурой. Таким образом, температуры газов определяют направление возможного самопроизвольного перехода тепла между ними без какого либо
дополнительного процесса, связанного с затратой определенной работы.
Температура характеризует состояние газа независимо от его мас-
Рис. 4.2 Определение полного давления трубкой Пито
сы, поэтому она является параметром состояния.
9
Измерение температуры можно производить только косвенным пу-
Плотность (ρ) это величина, определяемая для однородного ве-
тем, основываясь на зависимости от температуры таких физических свойств
щества (и средняя плотность неоднородного) его массой в единице объема
вещества, которые поддаются непосредственному измерению (н а п р и м е
 êã 
 3  (4.2).
ì 
р, с помощью термометра). Поскольку физические свойства веществ в
большей или меньшей степени зависят от температуры, то в качестве тер-

мометра может быть использован прибор, основанный на точном, легко воспроизводимом измерении какого либо свойства вещества (н а п р и м е р -
точке это предел отношения массы среды к её объему, когда объем стяги-
В настоящее время наиболее употребительной в приборах для из-
вается к этой точке.
мерения температуры является международная сто - градусная темпера-
Плотность среды, как правило, уменьшается при увеличении тем-
 
турная шкала (шкала Цельсия C ). Эта шкала имеет интервал температур
пературы и увеличивается при увеличении давления.
от точки плавления льда до точки кипения воды при атмосферном давлении
При переходе вещества из одного агрегатного состояния в другое
и разбита на сто равных частей (градусов).
(н а п р и м е р:
Нулевая точка абсолютной температуры по шкале Кельвина соот-
лед в воду, вода в пар и наоборот) плотность меняется
скачкообразно: резко уменьшается при переходе в газообразное состояние
ветствует минимальной теоретически возможной температуре (абсолютный
и, как правило, увеличивается при затвердевании.
ноль температуры). Таким образом, шкалы Кельвина и Цельсия просто
Плотности воды и чугуна аномально уменьшаются при переходе из
смещены друг относительно друга, и величины одного градуса у них равны.
жидкой фазы в твердую фазу.
В связи с тем, что в уравнения газовой динамики при решении за-
Для идеального газа плотность определяем по уравнению состоя-
дач необходимо подставлять температуру по шкале Кельвина, то темпера-
ния (4.3):
туру, заданную по шкале Цельсия, необходимо переводить в шкалу Кельвина по формуле (4.1).

K 
4.2
Плотность неоднородной среды, определяемая в определенной
хромель-копелевая термопара и т.п.).
T  t C  273,15
m
V
4.1
P
R T
4.3
где: Р – давление; R –газовая постоянная;
T – абсолютная температура.
4.3. Плотность
Плотность сухого газа, имеющего при определенных (нормальных)
условиях плотность ρн , при давлении Р и температуре Т определяется следующим образом (4.4):
10

 í  Ð  Òí

4.4
Ðí  Ò  ê
p mg

  g
V
V
4.6
где: р – вес; V – объем вещества; g – ускорение свободного паде-
где: ρн – плотность сухого газа при известных (нормальных) условиях;
ния;
Рн и Тн – давление и температура газа (соответственно) при которых извест-
m – масса; ρ – плотность.
на величина ρн; ρ – определяемая плотность сухого газа при определенных
условиях Р и Т; к – коэффициент сжимаемости, характеризующий отклонение реального газа от идеального.
4.5. Удельный объем
Плотность влажного газа (4.5)

 í  Ð    Pâ   Òí
Ðí  Ò  ê
 â   â
Удельный объем (V) газа представляет собой объем, занимаемый
4.5
ì
единицей плотности вещества 
3
 . Удельный объем связан с массой те êã 
где: φ – относительная влажность газа; Рв, φв и ρв - табличные зна-
ла и его объемом, следующим соотношением (4.7)
чения максимально возможного давления при заданной температуре, максимально возможной влажности и плотности водяного пара при заданном
V
давлении и температуре.
При экспериментальном определении плотности газа используют
зависимость его плотности от скорости распространения в нем звуковых
V
m
4.7
4.6. Вязкость
волн, или от интенсивности разного рода излучений прошедших через него.
Под вязкостью (внутренним трением) понимают свойство газа
оказывать сопротивление перемещению его слоев. Физической причиной
4.4. Удельный вес
вязкости является молекулярное взаимодействие. В газах вязкость есть результат взаимодействия, обусловленный хаотическим движением молекул.
 í 
Удельный вес (γ) – отношение веса вещества к его объему, 
.
ì 3 
Поэтому при повышении температуры в газах вязкость увеличивается за
счет более интенсивного движения молекул (в отличие от жидкостей, где
В соответствии со вторым законом Ньютона, плотность и удельный
повышение температуры приводит к снижению вязкости, т.к. происходит
вес вещества связаны между собой соотношением (4.6).
увеличение среднего расстояния между молекулами).
В условиях установившегося ламинарного движения, при постоянной температуре, вязкость газов не меняется. Она не зависит от плотности
11
или давления, а при нагревании - пропорциональна изменению температу-
чина силы трения, возникающая между слоями движущегося газа, опреде-
ры. Вязкость является функцией только абсолютной температуры.
ляется по формуле, предложенной Ньютоном и подтвержденной многочис-
Рассмотрим течение вязкого газа между двумя параллельными
ленными опытами других исследователей (4.8).
пластинами (рис. 4.3). При небольших скоростях движения газа векторы ско-
c  c1
dc
F  2
S 
S
y 2  y1
dy
ростей его частиц параллельны пластинам (такое течение носит название
ламинарного).
Y
где:
(4.8)
c2  c1
- градиент скорости течения (быстрота изменения скоy 2  y1
рости течения от слоя к слою), иначе скорость сдвига или поперечный гра-
b
а
диент скорости;  - коэффициент пропорциональности (динамическая вяз-
сmax
кость), он характеризует сопротивление газа смещению слоев; S - пло-
с+dc
с
щадь слоя по которой происходит сдвиг.
b
dy
A
dc
Разделим обе части уравнения (4.8) на S. Отношение силы трения
а
X
на площадь есть не что иное, как касательное напряжение τ (4.9).

Рис. 4.3. Эпюра скоростей газа при его движении в круглой трубе
Из-за наличия сил трения скорости частиц газа, находящихся на
F
dc

S
dy
(4.9)
Можно сказать, что вязкость газа - это способность его оказывать
стенках, равны нулю и возрастают по мере приближения к середине между
сопротивление касательным напряжениям.
пластинами (сmax) как это показано на рис. 4.3.
Из (4.9) можно сделать еще один важный вывод. Если газ находится
Выделим два слоя газа (a-a и b-b), расположенных на расстоянии
в состоянии покоя, то с=0 и, следовательно, τ=0, т.е. в покоящемся газе си-
dy. Пусть слой a-a движется со скоростью с, тогда, как следует из рисунка
лы вязкости не проявляются.
4.3, слой b-b имеет скорость с+dс.
Равновесное состояние сплошной среды характеризуется распре-
Разность скоростей на верхней и нижней гранях прямоугольной ча-
делением ее параметров в пространстве. Иначе в веществе начинает про-
стицы газа (выделена окружностью и обозначена А рис. 4.3), расположенной
исходить механический или тепловой обмен, который стремится сгладить
между слоями указывает на ее деформацию. Взаимодействие молекул че-
неравномерность. Этот обмен называют процессом переноса. В различных
рез этот элемент приводит к появлению касательной составляющей напря-
явлениях можно наблюдать процессы переноса энергии, массы (вещества) и
жения. При этом ее направление, таково, что оно соответствует уменьше-
количества движения.
нию разности скоростей по обе стороны рассматриваемого элемента. Вели-
12
Для определения, каким процессом переноса количества движения
Разделим обе части уравнения 4.10 на dt  dy 
обусловлена вязкость, рассмотрим формулу (4.9) относительно размерностей физических величин, входящих в нее
dc
dl

dy dt  dy
ì 

êã 

 Í   êã  ì  
   Ïà    2    2 2    ñåê2 
 ì   ñåê  ì   ñåê  ì 


Учитывая то, что
В числителе (произведение массы на скорость) - количество движе-
4.11
dl
 tg , получим
dy
dc
1
 tg
dy dt
ния, т.е. τ - это количество движения, переносимое через единицу поверхно-
4.12
сти в единицу времени. Следовательно, вязкость обусловлена процессом
переноса количества движения.
Следовательно, поперечный градиент скорости представляет собой
Для установления физического смысла скорости сдвига, рассмот-
скорость относительной деформации сдвига. Таким образом, касательное
рим частицу газа (обозначена буквой "А"), показанную на рис. 4.3. Из-за раз-
напряжение в газе линейно зависит от скорости относительной деформации.
ности скоростей на верхней и нижней гранях, первоначально прямоугольная
В этом принципиальное отличие газа от твердого тела, в котором касатель-
частица будет деформироваться, и превращаться в параллелограмм (рис.
ные напряжения зависят от величины деформации, а не от ее скорости.
4.4).
Если газы (жидкости) подчиняются уравнению (4.9) их называют
ньютоновскими, в противном случае - неньютоновскими.
dl
4.6.1. Динамическая вязкость
с+dc

ной силе приходящейся на единицу площади, необходимой для поддержа-
с
ния разности скоростей равной единице между двумя параллельными слоями газа, расстояние между которыми равно единице.
X
Динамическая вязкость зависит от физической природы газа, его
агрегатного состояния и абсолютной температуры, и практически не зависит
Рис. 4.4. Деформация частицы газа
от давления.
Для определения вязкости существует ряд формул.
Отрезок dl характеризует величину деформации за время dt
dl  dc  dt

Динамическая вязкость (   Ïà  ñ ) численно равна тангенциаль-
dy
Y
Формула Саттерлэнда (4.13)
4.10
13
Для очень разряженных газов понятие вязкости теряет смысл.
3
  T  2 T0  Ts
 
0  T0  T  Ts
4.13
При экспериментальных исследованиях вязкость определяют вискозиметрами. Наиболее распространены капиллярные, ротационные, с ша-
где: Тs – постоянная Саттерлэнда (имеет для воздуха значение
риком, ультразвуковые вискозиметры.
близкое к 114 К), Т0 и η0 – абсолютная температура и коэффициент вязкости,
Определения вязкости капиллярным вискозиметром состоит в из-
соответствующий некоторому начальному состоянию газа.
мерении времени протекания известного количества газа через узкие трубки
Широко также применяется степенная формула (4.14)
круглого сечения (капилляры) при заданном перепаде давления.
 T 
 
0  T0 
n
В ротационных вискозиметрах газ движется между двумя цилин-
4.14
драми, причем один из них вращается, а другой нет. Вязкость определяется
по крутящему моменту при заданной угловой скорости или по угловой скоро-
где:
n=1/2
при
Т>600
К;
n=1
при
Т<90К;
n=8/9
сти при заданном крутящем моменте.
при
Действие вискозиметра с шариком основано на определении вязко-
90К<Т<300К;
сти по скорости прохождения шариком промежутков между метками или от-
n=3/4 при 250К<Т<600К.
клонение шарика от вертикали при движении газа.
 1 
Величина обратная вязкости – текучесть (φ) 
 (4.15)
 Ïà  ñ 

1
Работа ультразвуковых вискозиметров основана на измерении скорости затухания пластины, погруженной в газ.
4.15

4.7. Сжимаемость
4.6.2. Кинематическая вязкость
Сжимаемость – способность вещества изменять свой первоначальный объем под воздействием перепада давления или при изменении
В технических приложениях часто используется не динамическая, а
температуры.
кинематическая вязкость (v), представляющая собой отношение динами-
ì
ческой вязкости к плотности среды 
Хотя сжимаемость (или ее обратная величина – упругость) является
2
 (4.16)
свойством, которое, строго говоря, выводит нас за рамки газовой динамики,
 ñ 
ее, по крайней мере, при упрощенной постановке задачи, приходится учитывать по соображениям двоякого рода. Во-первых, реальные газы представ-

v

ляют собой упругие среды, в которых распространяются звуковые волны
4.16
(скорость звука в воздухе при нормальны условиях составляет 335 м/с.). Н а
п р и м е р , если течение газа в трубопроводе резко перекрыть краном или
14
задвижкой, то возмущение от остановки течения будет распространяться
где: с – скорость движения газа; а – местная скорость звука.
вверх по трубопроводу со скоростью звука, причем уменьшение скорости
среды позади такой волны возмущения будет сопровождаться заметным повышением давления. Во-вторых, сжимаемость приходится учитывать и по
До тех пор, пока величина М мала, влияние сжимаемости незначи-
той причине, что именно этим свойством определяется возможность анали-
тельно. Когда же число Маха приближается к единице, картина течения су-
за жидкости и газа на основе одних и тех же принципов. Критерием при этом
щественно изменяется в связи со звуковыми эффектами.
Н а п р и м е р , коэффициент лобового сопротивления пули со
служит отношение скорости течения к скорости упругой волны, т.е. к скоро-
сферической головной частью зависит только от числа Рейнольдса до тех
сти звука в данной среде.
Сжимаемость газа становится существенной при больших скоростях
пор, пока число Маха не превысит 0,5. При дальнейшем разгоне коэффици-
движения среды соизмеримых со скоростью распространения звука в этой
ент лобового сопротивления возрастает и приблизительно удваивается, по-
среде и больше, т.к. при таких скоростях возникают большие перепады дав-
сле того как М становится больше единицы. Это происходит вследствие об-
ления и температуры. Величину сжимаемости характеризует коэффициент
разования звуковых волн (скачков уплотнения) в зоне сжатия непосред-
сжимаемости  . Этот коэффициент выражает уменьшение единичного
ственно перед пулей.
На рис. 4.5 представлены фотографии, выполненные с помощью
объема (или увеличение плотности) газа при увеличении давления на еди-
двух различных оптических методов. На них показаны волновая картина и
ницу измерения (4.17).
след винтовочной пули, летящей в атмосферном воздухе со скоростью, не-
1  V  1   
  
 
;
V  P    P 
 1
ì 2



í 
 Ïà
сколько превышающей скорость звука. Различия в изображении проявились
из-за того, что показанная вверху фотография получена теневым методом,
4.17
чувствительным к изменениям второй производной плотности газа. Нижняя,
Знак "минус" показывает, что увеличение давления сопровождается
шлирен-фотография, показывает градиенты плотности по нормали к кромке
уменьшением объема и наоборот.
ножа, которая здесь вертикальна.
Подобно тому, как носовой части быстроходных судов придают заостренную и тщательно спрофилированную форму для уменьшения носовой
4.8. Число Маха
волны и, следовательно, волнового сопротивления, заостряют носовые части высокоскоростных снарядов, передние кромки крыльев самолетов и
Число Маха – основная характеристика движения газа, представ-
кромки лопаток рабочих колес сверхзвуковых турбин.
ляющая собой отношение скорости газа к местной (в данной точке среды)
Это делается для того, чтобы уменьшить потери в скачках уплот-
скорости звука (4.18).
нения, а тем самым уменьшить сопротивление, связанное со звуковыми
эффектами.
c
M
a
4.18
15
4.9. Энтальпия
 êÄæ
Энтальпия (теплосодержание) (h), 
 – термодинамический
 êã 
потенциал, характеризующий состояние макроскопической системы в термодинамическом равновесии при выборе в качестве основных независимых
переменных энтропии и давления.
Другими словами – энтальпия это состояние термодинамической
системы, равное сумме внутренней энергии и произведения давления на
объем системы (4.19).
h  U  pV
Рис. 4.5 Волновая картина и след винтовочной пули
4.19
где: U - внутренняя энергия; p - давление; V - объем системы.
О больших энергетических потерях, обусловленных образованием
Для идеальных газов определить энтальпию можно по формуле
звуковых волн, можно судить по тому шуму, который создают воздушные
(4.20)
винты самолетов, и по пронзительному звуку, которым сопровождается по-
h  c pT
лет снарядов и ракет.
В связи с вышеизложенным, число Маха является одним из основ-
4.20
где сР – изобарная теплоемкость газа, которую можно определить
ных газодинамических характеристик потока и является мерой влияния
(4.21)
сжимаемости газа на его движение.
cp 
При М<<1 газ можно считать несжимаемым.
k
R
k 1
4.21
Для воздуха сжимаемость необходимо учитывать при М>0,3.
где k – коэффициент изоэнтропы (н а п р и м е р для воздуха k=1,4);
В зависимости от значения числа Маха движение потока газа или
движение объекта в газовой среде разделяют на следующие типы:
 Äæ 
R – Газовая постоянная 
 , зависящая от природы газа
 êã  K 
• - если число Маха меньше единицы, такое течение - дозвуковое;
• - при числе Маха равном единице – течение звуковое;
 Äæ 
• - если число Маха больше единицы – течение сверхзвуковое;
(н а п р и м е р для воздуха R=287 
 ) и определяемая как отношение
 êã  K 
• - при числе маха больше пяти – течение гиперзвуковое.
универсальной газовой постоянной к молярной массе газа.
16
Физический смысл газовой постоянной R – она равна работе, за-
Из уравнения 4.23 видно, что при:
траченной на расширения 1 кг конкретного газа при нагреве его на один гра-
•  S =0 – процесс обратимый, может протекать как в прямом, так и
дус в изобарном процессе.
в обратном направлении (идеальный случай, на практике не достижим);
•  S >0 – процесс необратимый, самопроизвольно протекает только в одном направлении (н а п р и м е р: увеличение энтропии потока за счет
4.10. Энтропия
повышения его статической температуры из-за трения);
•  S <0 – такой процесс не может протекать произвольно, необхо-
 Äæ 
Энтропия – S 
 - функция состояния термодинамической си êã 
дим подвод энергии извне (н а п р и м е р: работа холодильных машин).
стемы, характеризующая направление протекания процесса теплообмена
• - поскольку величина температуры всегда положительна, из этого
между системой и внешней средой, а также направление протекания само-
следует, что при подводе тепла к системе ее энтропия возрастает, а при от-
произвольных процессов в замкнутой системе.
ведении убывает.
При определении изменения энтропии для идеальных газов можно
В связи с тем, что внутри системы, выполняющей какую либо рабо-
воспользоваться следующими формулами (4.24 или 4.25):
ту, происходят необратимые процессы преобразования энергии в тепло, сопровождаемые потерями энергии, энтропия является мерой необратимого
T
P
S 2  S1  c p ln 2  R ln 2
T1
P1
рассеивания энергии. Определить ее можно следующим образом (4.22)
Q  Q2 dQ
dS  S1  S2  1

T
T
S2  S1  cv ln
4.22
где Q – количество теплоты, полученное или отданное газом;
P vk
P2
v
 c p ln 2  cv ln 2 2
 Pvk
P1
v1
 1 1
4.24




4.25
Все процессы в природе протекают в направлении увеличения эн-
Т – абсолютная температура.
тропии.
Поскольку на практике используют обычно не саму величину энтро-
Вопросы для самопроверки
пии, а ее изменение, нулевое значение энтропии для газа или смеси газов,
не вступающих между собой в химическую реакцию, может быть выбрано
1.
Какие силы, действующие в газе, понимают под объемными силами?
произвольно. Это возможно благодаря тому, что энтропия является функци-
2.
Какие силы, действующие в газе, понимают под поверхностными силами?
ей состояния и, следовательно, изменение ее в каком либо процессе опре-
3.
Что понимают под понятием экстенсивные параметры состояния системы?
деляется разностью энтропий в точках начала и конца процесса и не зави-
4.
Что понимают под понятием интенсивные параметры состояния системы?
сит от выбора начала отсчета самой энтропии (4.23).
5.
Что такое плотность газа?
6.
В каких единицах измеряется плотность газа?
7.
Как зависит плотность газа от температуры?
2
dQ
S  S2  S1  
1 T
4.23
17
8.
Что такое удельный вес?
9.
В каких единицах измеряется удельный вес газа?
5. Закон сохранения массы
Массой тела называют физическую величину, являющуюся мерой
10. Как зависит удельный вес газа от температуры?
11. Что такое вязкость газа?
его инерционных и гравитационных свойств.
12. В каких единицах измеряется вязкость газа?
Между энергией и массой вещества существует прямая связь. Каж-
13. Как зависит вязкость газа от температуры?
дому изменению массы соответствует определенное изменение энергии и
14. Что такое динамическая вязкость газа?
наоборот. Поэтому так же как соблюдается закон сохранения энергии (о ко-
15. Чем отличается динамическая вязкость газа от кинематической вязкости
тором речь пойдет ниже) должен соблюдаться закон сохранения массы ве-
газа?
щества, в том числе и газа.
16. В чем состоит свойство сжимаемости газа?
Закон сохранения массы гласит - масса изолированной системы не
17. Что представляет собой такая характеристика потока газа как число Маха?
изменяется при любых происходящих в ней процессах.
18. Что такое давление газа?
В Ньютоновской механике (скорости движения во много раз меньше
19. В каких единицах измеряется давление газа?
скорости света) считается, что масса объекта не зависит от скорости движе-
20. Что такое статическое давление газа?
ния.
21. Что такое динамическое давление газа?
В связи с тем, что газ является сплошной средой, которая движется
22. Что такое энтальпия?
в канале (соплах, трубах, щелях и т.п.), а стенки канала непроницаемы, яс-
23. Что такое энтропия?
но, что в единицу времени через любое сечение канала протекает одинаковое количество газа. Другими словами: масса газа, протекающего через любое сечение непроницаемой трубы – величина постоянная и называется
 êã 
расходом газа   (5.1)
ñ
G
m

 const ,
5.1
где: m – масса газа; τ – время.
Рассмотрим канал переменного сечения (рис. 5.1) по которому течет газ. Движение газа считаем установившимся.
На основании закона сохранения массы и условия неразрывности
течения для установившегося течения можно считать, что масса газа, вошедшая в канал равна массе газа вытекающего из него в единицу времени.
18
В связи с этим, скорость движения частиц, проходящих через канал меньше-
ставляющего собой закон сохранения массы (в некоторых источниках техни-
го сечения должна увеличиваться.
ческой литературы его называют уравнением сплошности).
G
F1
m


 F S
   F  c  const , кг/с

5.5
F2
Рассмотрим изменение скорости газа в канале с расширяющейся
частью – диффузоре (рис. 5.2).
S2
S1
c1
c2
Рис. 5.1. Канал переменного сечения
F1 и F2 входная и выходная площади (соответственно);
S1 и S2 расстояние пройденное частицами газа в широкой части
и узкой части канала (соответственно) за одну единицу времени.
Рис. 5.2. Диффузор
Скорость газа (5.2)
c
S

, м/с
с1 – скорость газа на входе; с2 – скорость газа на выходе.
5.2
Запишем уравнение неразрывности 5.5 в следующем виде (5.6)
Масса газа (5.3)
m    V , кг
5.3
1c1F1  2c2F2
Объем газа (5.4)
V  F  S , м3
5.6
При невысоких скоростях (меньше 0,3 от числа Маха) газ можно
5.4
считать несжимаемым, то есть ρ1= ρ2, а площадь на выходе больше площади на входе в диффузор. Следовательно, для сохранения равенства (5.6)
Подставив уравнение 5.4 в уравнение 5.3, а затем в уравнение 5.1,
скорость газа на выходе должна быть меньше чем скорость газа на входе в
учитывая уравнение 5.2, получим уравнение неразрывности (5.5), пред-
диффузор (с2<с1).
19
6. Закон сохранения энергии
Рассмотрим изменение скорости газа в канале с суживающейся частью – конфузоре (рис. 5.3).
Для правильной оценки возможностей тепловой машины (н а п р и м
е р, турбины) необходимо уметь определить ее мощность и кпд. Для этого
необходимо иметь четкое представление, каким образом подведенная энер-
c1
c2
гия превращается в механическую работу и другие виды энергии. Закон сохранения энергии позволяет связать газодинамические процессы с одновременно протекающими энергетическими превращениями.
Энергией называется способность объекта совершать работу. Это
общая количественная мера движения и взаимодействия всех видов материи.
Закон сохранения и превращения энергии звучит следующим обра-
Рис. 5.3. Конфузор
зом – при любых процессах, происходящих в изолированной системе, ее
полная энергия не изменяется.
с1 – скорость газа на входе; с2 – скорость газа на выходе.
В силу того, что движение может иметь разнообразные формы, существуют различные формы энергии (кинетическая, внутренняя, потенци-
В связи с тем, что в данном случае площадь канала на входе боль-
альная). Если рассматриваемая система не меняет своего состояния, то
ше, чем на выходе из конфузора, используя выше приведенные рассужде-
численное значение суммы всех видов энергии остается неизменным при
ния видно, что скорость газа на выходе возрастет в сравнении со скоростью
всех взаимопревращениях движений.
газа на входе (с1<с2).
Изменить состояние системы можно с помощью внешних сил, совершающих работу над системой (заставить систему совершить работу),
Обратить внимание – поскольку для потоков газов с околозвуко-
или с помощью теплообмена.
выми и сверхзвуковыми скоростями сжимаемость оказывает очень сильное
Рассматривая изменение полной энергии системы, закон сохране-
влияние на плотность все это справедливо только для газов, у которых
ния энергии можно представить так: приращение полной энергии энергети-
плотность не меняется, то есть для потоков газа с очень малыми скоростями
чески неизолированного объема газа за единицу времени равно работе, ко-
(меньше 0,3M).
торую внешние силы совершили над ним и притоку теплоты за ту же единицу времени.


Другими словами - энергия (Е) (единицы измерения которой Äæ ;
Í
 ì ; Âò  ñ ) не возникает из ничего и не исчезает, она может перехо-
дить из одной формы в другую.
20
W+Eвн=Еп+Ек+Евнут.+Lм+ Lтр.
К газу энергия подводится в виде тепловой энергии (повышение
6.1
температуры газа за счет другого объекта посредством теплообмена) и в
где: W – тепловая энергия; Eвн – энергия внешних сил; Еп - потенци-
виде работы внешних сил (давление, гравитация).
альная энергия; Ек – кинетическая энергия; Евнут – внутренняя энергия; Lм -
При рассмотрении процессов, происходящих в газовых турбинах и
работа механическая; Lтр. – работа сил трения.
компрессорах, полная энергия состоит из:
• кинетическая энергия - энергия системы, зависящая от скорости
Проанализируем бесконечно малое изменение энергии бесконечно
ее частиц;
малой массы газа:
• потенциальная энергия - часть общей энергии системы, завися-
• dEп=dG(z2-z1) – в рассматриваемых условиях течения газа, харак-
щая от взаимного расположения материальных точек, составляющих эту си-
терных для установок, эксплуатирующихся в машинных отделениях судов
стему и от их положения во внешнем силовом поле;
(трубопроводы, лопаточные машины ит.п.) - (z2-z1)≈0 , поэтому изменением
• внутренняя энергия - энергия системы, зависящая только от ее
потенциальной энергии можно пренебречь, то есть dEп=0;
термодинамического состояния;
• dЕвнут.=dG(E2-E1), известно, что Е=сvT, следовательно, если теп-
• механическая работа - работа, снимаемая с вала турбины или
лоемкость газа (сv) не изменяется, то можно записать
подведенная к валу компрессора;
dЕвнут.= сvdG(Т2-Т1)
• работа сил трения возникает в пограничном слое.
При составлении уравнения баланса энергии в неподвижной си-
• Eê 
стеме координат (6.1) рассматриваем изменение энергии в одной и той же
6.2
m  c2
приращение любого вида энергии, в том числе и ки2
массе газа, заполнявшего в первоначальный момент времени один объем
нетической равно разности количеств этого вида энергии в положениях 11-21
(1-2), а в процессе движения, за бесконечно малый промежуток времени, за-
и 1-2 согласно рис. 6.1.
полнившим другой объем (11-21) согласно рис. 6.1.
Ввиду того, что заштрихованный объем 11-2 является общим для
этих двух положений, энергия массы газа, заполняющей объем 11-2, при вы-
21
2
читании сокращается, так как движение газа считаем установившимся. Приращение энергии измеряется разностью количеств энергии в бесконечно
малых объемах 2-21 и 1-11 следовательно, приращение кинетической энергии равно (6.3)
11
c 2  c12
dE ê  dG  2
2
1
6.3
• Рассмотрим dEвн . Силы гравитации для газов не учитываются, так
как они практически не влияют на процесс его движения, однако на выдеРис. 6.1. Элементарная струйка газа
ленную струйку газа действуют направленные внутрь и по нормали к ним
21
внешние силы давления. При перемещении газа внешние силы производят
2
2
dL òð
dL
dW  P1 P2  c 2  c1
 
 

 c v T2  T1   ì 
dG  1  2 
2
dG
dG
работу, поэтому изменение энергии внешних сил можно записать так (6.4)
согласно рисунку 6.1. Если Р1 – давление, действующее на площадку F1 и
приводящее газ в движение, то оно положительное, а вот Р2 - это давление
Рассмотрим составляющие уравнения (6.8).
сопротивления движению газа, значит, оно направлено всегда против движения и имеет знак минус.
dЕвн.=Р1F1c1dτ - Р2F2c2dτ
Из курса термодинамики известно, что
нение расхода газа в единицу времени
В итоге уравнение (6.8) можно записать в следующем виде (6.9)
dG    F  c  d
или

 F  c  d
dW
 Q - тепло.
dG
dLì
 Lì - работа механическая отнесенная к 1 кг газа.
dG
dLòð
 Lòð - работа сил трения отнесенная к 1 кг газа.
dG
6.4
Воспользовавшись уравнением неразрывности (5.5), зададим изме-
dG
6.8
2
2
P
P  c  c1
Q   1  2   2
 c v T2  T1   Lì  Lòð
2
 1  2 
6.5
6.9
В общем случае тепло ( Q ) к массе газа может подводиться снару-
Подставив уравнение (6.5) в (6.4), получим
жи через поверхность канала, а также появляется внутри за счет трения га-
P
P 
dE âí  dG   1  2 
 1  2 
за.
6.6
Q  Qíàð  Qâí
Подставляем уравнения (6.2), (6.3) и (6.6) в уравнение баланса
Вспомним уравнение состояния идеального газа
энергии (6.1) получим (6.7)
c 2  c12
P
P 
dW  dG  1  2   dG 2
 c v dG T2  T1   dL ì  dL òð
2
 1  2 
6.10
P  V  R T
6.11
6.7
где: R=cp-cv – газовая постоянная; cp – изобарная теплоемкость; cv изохорная теплоемкость; Р – давление; V – удельный объем.
Разделим обе части уравнения (6.7) на dG
22
Энтальпия (h) – функция, отражающая теплосодержание термоди-
c 2  c12
Q  Lì  2
 h2  h1
2
намической системы, равная сумме внутренней энергии системы и произве-
6.17
дения давления на объем (6.12)
h  Eâíóò  P  V
6.12
Если газ движется, не совершая механическую работу (L=0) и нет
теплообмена с внешней средой (процесс адиабатный Q=0), то
При постоянном давлении приращение энтальпии равно количеству
теплоты сообщенной системе (6.13)
h  c pT
c 2  c12
 h2  h1  2
2
6.13
6.18
или
В связи с тем, что ср=сv+R уравнение 6.13 можно записать в следу-
c2
c2
h2  2  h1  1
2
2
ющем виде (6.14)
h  cv  T  R  T  cv  T 
6.19
P
При подводе теплоты или работы к газу - теплоту и работу считают

со знаком "минус". При совершении работы газом ее рассматривают со зна-
6.14
ком "плюс".
Следовательно (6.15)
P
P
h2  h1  cv  T2  2  cv  T1  1
2
1
6.15
Если газ проходит через проточную часть турбины, то есть совершается работа, то он расходует энергию (Lтурб>0). В случае прохождения га-
Или (6.16)
cv  (T2  T1)  h2 
за через компрессор, он получает энергию (Lкомпр<0). Уравнение (6.17) мож-
P2
2
 h1 
P1
1
но записать в виде (6.20) для турбины и (6.21) для компрессора.
6.16
Уравнение (6.16) подставим в (6.9) и уберем из него работу трения.
Это возможно сделать, поскольку энергия, расходуемая на преодоление
любого вида сопротивления, преобразуется полностью в тепло, а последнее
остается в струе газа. То есть наличие сил трения не нарушает общий баланс энергии, а приводит к преобразованию одного вида энергии в другой. В
итоге получаем уравнение (6.17).
P
P  c 2  c12
P
P
Q   1  2   2
 h2  2  h1  1  Lì
2
2
1
 1  2 
c 2  c12
h2  h1  2
 Lòóðá
2
6.20
c2  c2
h2  h1  2 1  Lêîìïð
2
6.21
Для машин, работающих на сжатие, затрачивается внешняя работа,
следовательно, в массу газа вводится дополнительная энергия, из-за чего
температура газа повышается.
или
23
В турбинах рабочее тело само совершает работу, следовательно,
где: T * - температура торможения, T - статическая температура; с
часть энергии газа отбирается, из-за чего температура газа падает. На ис-
– скорость потока; сР – изобарная теплоемкость.
пользовании этого свойства основана работа турбодетандеров (холодиль-
Если температуру торможения можно определить с помощью обыч-
ных машин).
ного термометра или термопары, то статическую температуру измерить с
При рассмотрении адиабатного течения потока газа, не совершаю-
достаточной точностью очень проблематично, так как в этом случае термо-
щего работы, уравнение (6.19) можно записать следующим образом (6.22)
метр должен лететь вместе с потоком. Поэтому для ее определения используют формулу (6.24), предварительно измерив температуру торможения,
h2 
c22
2
 h1 
c12
2
h
2
c
 const
2
скорость потока и по справочным таблицам определив величину изобарной
6.22
теплоемкости.
Из уравнения (6.22) не трудно заметить, что теплосодержание газа
Уравнение энергии можно записать и в другом виде.
(энтальпия) достигает своего максимального значения при скорости потока
газа равной нулю.
Для этого воспользуемся уравнением состояния совершенного газа
(газ удовлетворяющий условию P    R  T ), и, учитывая, что R  c p  cv
Другими словами – если струю газа затормозить до нулевой скорости, то теплосодержание его достигнет максимума. В этом случае все пара-
получим
метры называют параметрами торможения (в дальнейшем параметры
P
торможения будут обозначаться с верхним индексом "*"). Уравнение сохра-

нения энергии (6.22), в этом случае, можно записать (6.23).
h*  h 
c2
 const
2
 (c p  cv )  T
Обе части уравнения 6.25 разделим на сv
6.23
P
  cv

(c p  cv )
cv
В этом уравнении h * - эквивалентно полной энергии, h - эквивалентно внутренней энергии, а
c2
- кинетическая энергия.
2
 cp

 T  
 1  T  (k  1)  T
c
 v

6.26
где; k=cp/cv – коэффициент изоэнтропы
Учитывая, что h  c p  T уравнение сохранения энергии (6.23) мож-
из уравнения 6.26 следует, что
но записать в виде (6.24)
T 
2
c
T* T 
 const
2  cp
6.25
6.24
24
P
  cv  (k  1)
6.27
Подставив выражение 6.27 в уравнение теплосодержания h  c pT ,
В том случае, если число Маха (отношение скорости тела к местной
скорости звука) равно 10, нагрев доходит до 6000°С, то есть до температу-
получаем
ры, при которой испаряются все известные в природе вещества.
P
k P
k
h  c pT  c p

 
 R T
  cv  (k  1) k  1  k  1
Эти числа позволяют представить, что происходит, когда космические тела или корабли входят в плотные слои атмосферы. Малые тела (ме-
6.28
теоры) - сгорают, не достигнув поверхности земли. Крупные тела (метеориты) – оставляют в воздухе большую часть своей массы.
Из уравнения 6.28 видно, что
k
cp 
R
k 1
Проблема возвращения космического корабля на Землю являлась
серьезным препятствием на пути освоения космоса. Все первые спутники
6.29
Земли существовали до тех пор, пока находились достаточно далеко от границы атмосферы, при приближении к Земле они сгорали. По этой причине
Тогда уравнение сохранения энергии будет выглядеть в виде (6.30)
современные космические корабли оборудованы двигателями торможения,
или как (6.31)
P c2
P
c 2
k
k
 1 1 
 2  2  const
k  1 1
2
k  1 2
2
однако для полной остановки корабля потребовалось бы столько же топлива, что и для разгона. Поэтому торможение осуществляется на некоторую
6.30
величину. При этом корабль сходит со своей околоземной орбиты и входит в
или
атмосферу, имея уже меньшую скорость. Однако он все равно подвергается
c2
c 2
k
k
k
 R  T1  1 
 R  T2  2  const 
 R  T * 6.31
k 1
2
k 1
2
k 1
аэродинамическому нагреву до температур, превосходящих температуру
испарения материала из которого изготовлен корпус.
Во избежание аварии носовую часть корабля покрывают толстым
Используя уравнение (6.24) с учетом (6.29) можно получить изоэн-
слоем вещества с малым коэффициентом теплопроводности, высокой тем-
тропийную (теоретическую) скорость потока газа (6.32).
c

2k
R T * T
k 1

пературой испарения и большой энергией испарения.
Применяемые покрытия при высоких температурах переходят из
твердого состояния в газообразное состояние, минуя жидкую фазу, и своими
6.32
испарениями защищают корабль от воздействия высокой температуры.
Перепишем формулу (6.32) в таком виде (6.33).
При анализе формулы (6.32), становится понятно, почему нагревается летящее с большой скоростью тело. При движении тела в воздухе со
c
скоростью звука температура воздуха непосредственно на поверхности тела
повышается, примерно на 60°С. Нагрев воздуха в пограничном слое сверхзвукового самолета ТУ-144 достигал около 400°С.
25

2k
T 
R  T * 1 

k 1
 T* 
6.33
7. Уравнение количества движения
При изоэнтропийном течении газа справедливо следующее соотношение (6.34) (вывод которого будет дан позднее)
К массе газа, заключенной в элементарном объеме применим вто-
T1
*
T0
k 1
 k
рой закон сохранения – теорема количества движения.
P
 1
 P* 
 0
Изменение количества движения массы газа происходит в общем
6.34
случае вследствие того, что каждая частица, перемещаясь, занимает с течением времени новое положение и приобретает новую скорость, а также
Для изоэнтропийного процесса, подставив значения отношений
потому, что ее скорость изменяется во времени.
температур из уравнения (6.34) в формулу (6.33), получаем возможность
При установившемся движении количество движения меняется
определить теоретическую скорость газа на выходе из сопла или канала,
только в связи с изменением положения частиц.
используя замеры температуры торможения, полного давления на входе в
В соответствии с упомянутой теоремой механики изменение коли-
сопло и статического давления на выходе из него (6.35).
чества движения массы, заключенной в выделенном элементарном объеме,
равно сумме импульсов действующих сил.
c12t
 c02
2

 P
k

R  T0 1   1
k 1
  P0
 





На элемент газа действуют внутренние и внешние силы.
k 1 
k 
Внешние силы создаются физическими полями, в пределах которых




формируется поток газа: гравитационным полем, полем механических сил,
силами инерции, в том числе центробежными или центростремительными
силами, и др.
6.35
Внутренние силы имеют иное происхождение, они возникают в
или (6.36)
движущемся газе благодаря молекулярному взаимодействию выделенной
c1t 

 P
2k
R  T0* 1   1
*
k 1
  P0

частицы с окружающим ее газом. К этой категории относятся поверхност-
k 1 
 k 






ные нормальные силы давления и касательные силы вязкости (трения).
6.36
Необходимо помнить - поверхностные силы пропорциональны площади поверхности, ограничивающей некоторый, мысленно выделенный объем газа.
Из курса физики известно, что количество движения (импульс) – это
где: c1t - теоретическая скорость газа на выходе из сопла; c0 скорость газа на входе в сопло; k – коэффициент изоэнтропы; R – газовая
произведение массы тела на его скорость. Изменение импульса тела постоянной массы может происходить только в результате изменения скорости, и
постоянная; T0* - температура торможения на входе в сопло; T0 - статиче-
всегда обусловлено действием силы (7.1).
ская температура на входе в сопло; P1 – статическое давление на выходе
из сопла; P0* - давление торможения на входе в сопло; P0 - статическое
P  mc
давление на входе в сопло.
где: P - вектор силы; m - масса объекта; c - вектор скорости.
26
7.1
Для решения задач газовой динамики рассмотрим в одномерном
го, что приращение давления на верхней и нижней границе струи равны и их
установившемся потоке элемент газа, выделенный из потока (рис. 7.1).
сумма равна полному приращению давления);
• - сила сопротивления (трения) dS , направленная вдоль боковой
На этот элемент газа действуют следующие силы:
• - в сечении F – сила давления ( P  F ) – она направлена слева
поверхности элемента, противоположно скорости потока.
Из второго закона Ньютона следует, что отношение величины силы,
направо;
• - в сечении F+dF – сила давления
действующей на тело, к приобретенному телом ускорению постоянно для


P 
dx F  dF  ,
 P 

x



данного тела. Это отношение представляет собой массу тела.
Поэтому сумма всех сил должна равняться произведению массы
направлена справа налево;
выделенного элемента потока газа на его ускорение (7.2) (Масса элемента
0
x, c
P
1 P
P+ 2 x
dm  F    dx ).
1
dx
Fdx
dS
F+dF
F
P+
P
x
dx
(7.3)
P+21 xP dx
Fdx
dx
1
dc
1 P
P
 
dxdF 
Fdx  dS
dt
2 x
x
7.3
Разделим все члены уравнения (7.3) на dm  F    dx
Рис. 7.1 К выводу уравнения количества движения
dc
1 P dxdF P Fdx
dS
 


dt
2 x Fdx x Fdx dm
с – скорость потока; Р – давление; S – сила сопротивления, отнесенная
или
к единице массы потока; F - площадь поперечного сечения элемента.

• - на боковую поверхность элемента – сила  P 

dc
1 P dF P 1 dS




 
dt
2   x F
x  dm
1 P  

dx dF  ,
2 x
 
Так как
равная проекции сил давления, перпендикулярных этой поверхности и
направленная слева направо (множитель
7.2
Раскрыв скобки, и сделав элементарные преобразования, получим
dS
0
dc
1 P
P




 P  F  P  
dx dF   P 
dx F  dF   dS
dt
2 x
x




1
P
перед
- появился из-за тоx
2
27
7.4
dF
 0 , уравнение (7.4) приобретает следующий вид (7.5)
F
Если рассматриваемый процесс идеальный, то есть при изоэнтроп-
dc
1 P
 
S
dt
 x
7.5
ном характере течения, можно найти плотность газа по уравнению изоэнтропы (вывод будет приведен в дальнейшем) (7.10)
где: S 
dS
dm
1
 P k
  0   
 P0 
сила сопротивления, отнесенная к единице массы
потока газа.
Подставив уравнение (7.10) в уравнение (7.9), получим (7.11)
Для одномерного, установившегося потока давление является
функцией одной переменной x и поэтому
dP P
, в связи с чем, уравне
dx
x
1
1
c  dc 
ние (7.5) будет выглядеть следующим образом
dc
1 dP
 
S
dt
 dx
7.10
1
k
P

 P0 
 0  
7.6
1  P0  k
 dP  c  dc 
    dP  0
0  P 
7.11
Обозначим скорость в сечении 0-0 с0 , а в сечении 1-1 с1t (теоретическая скорость, так как процесс изменения состояния между сечениями
был принят изоэнтропным).
Умножим левую и правую части уравнения на dx
В результате интегрирования уравнения (7.11) получим уравнение
dc
1 dP
 dx   
 dx  S  dx
dt
 dx
Учитывая то, что
c  dc  
1

количества движения (уравнение импульсов) для одномерных изоэнтропий-
7.7
ных потоков в интегральной форме (7.12, 7.13) или (7.14).
dx
 c , получаем уравнение (7.7) в виде (7.8)
dt
 dP  S  dx
c1t

 dP  0
c0
P0
1
P0k P0
0
имеет вид (7.9)
1

c  dc   
1
0
1
1
 P0  k

P0k
   dP  
0
P 
7.8
При отсутствии сил сопротивления (трения) S=0 и уравнение (7.8)
c  dc 
P1
7.9
28

dP
1
P1
Pk

1
P0k
0

1
 1
k
P
1
P1

dP
1
P0
Pk

P0
1
 1
k
P1

k 1 1

k
0
1
 P0k

P0k
0
P0
dP

P1
1
Pk
1
 1
k
P

P0
1
1
k
P1
k 1 1

k
0
1
k
 P0

1
 1
P k

8. Уравнение моментов количества движения
P0
1
1
k
P1

k 1 1

k
0
1
k
 P0
1
  1 1
 1 
 k
k
  P0
 P1





Из курса физики известно, что моментом количества движения системы материальных точек (из которых состоит газ) относительно полюса
называется вектор, равный геометрической сумме моментов количества
1
 1
k
Вынося P0
за скобку, получим (7.12)
движения относительно того же полюса всех точек системы.
Моментом количества движения системы материальных точек относительно оси называется скалярная величина равная проекции на ось
k 1 

c 2  c02
P0   P1  k 
k
1t



1  

2
k  1 0   P0 



вектора момента количества движения системы относительно любого полю7.12
са, принадлежащего этой оси.
Другими словами, моментом количества движения системы называется произведение ее момента инерции на угловую скорость. Эта величина
Используя уравнение состояния, формулу (7.11) можно записать в
является одной из мер механического движения системы.
следующем виде (7.13)
c 2  c02
1t
2
Изменение момента количества движения (при неизменном момен-
k 1 

  P1  k 
k

 R  T0  1   

k 1
  P0 



те инерции тела) может произойти только вследствие изменения угловой
скорости, и всегда обусловлено действием момента силы. Если на систему
7.13
не действуют моменты внешних сил (система замкнута), то ее полный угловой момент остается постоянным по величине и направлению.
Или для параметров торможения (7.14)
Другими словами – изменение суммарного момента количества
движения, относительно какой - либо оси, равно сумме моментов импульсов
k 1 
 k 

 P
2k
2
*
c 
 R  T0  1   1 
*
1t
k 1
  P0 




всех сил, приложенных к объекту, относительно той же оси.
Для вычисления момента количества движения материальной точки
7.14
относительно оси справедлива формула для вычисления момента силы,
если в ней заменить вектор силы вектором количества движения.
Рассмотрим установившееся движение элементарной массы газа
Обратить внимание - уравнение импульсов (7.13) для изоэнтро-
перемещающейся, за какой-то малый промежуток времени из объема 1-2 в
пийных потоков совпадает по форме с уравнением энергии (6.37). При не-
объем 11-21 (рис. 8.1).
изоэнтропийных потоках газа (потоков с необратимыми потерями энергии)
Если рассмотреть изменение момента количества движения отно-
уравнение энергии не изменяет своего вида, а уравнение количества дви-
сительно оси Y, то можно записать закон этого изменения в следующем ви-
жения должно включать член, учитывающий силы сопротивления (трения и
де (8.1):
т.п.).
29
d  m  c x  z  cz  x   MY  d
8.1
При установившемся течении
сократив подчеркнутые члены уравнения (8.2) и учитывая вышеизложенные
где: c x  m и c z  m - проекции количества движения элементарной
рассуждения, получаем уравнение (8.3).
массы m на оси Z и X; z и x – плечи приложения количества движения (соответствующие координаты); m  c x  z  cz  x  - момент количества движе-
d  mc x z  cz x   G  c2x z2  c2z x2   c1x z1  c1z x1  d
ния элементарной массы m относительно оси Y.
8.3
где: G – секундный расход газа.
cz 2 1
2
Z
dm
 G или dm  G  d , поэтому,
d
Учитывая формулу (8.1) - уравнение (8.3) можно записать в виде
(сократив d ), который используется в газовой динамике (8.4)
11
cy
1
cx
MY  G  c2x z2  c2z x2   c1x z1  c1z x1
Y
z
Аналогичным образом уравнения могут быть составлены для осей Х
x
y
8.4
и Z . Согласно уравнению (8.4) (уравнение Эйлера) сумма моментов относи-
X
тельно любой оси всех сил, приложенных к объему газа, равна разности
моментов относительно той же оси секундного количества движения выхо-
Рис. 8.1 К выводу уравнения моментов количества движения
дящего и входящего газа.
Так как мы рассматриваем установившееся движение газа, то изме-
Уравнение Эйлера, основанное на уравнении момента количества
нение его суммарного момента количества движения равно разности мо-
движения, удобно тем, что позволяет определять момент сил от потока газа,
ментов количества движения в элементарных объемах
объяс-
не вникая в сущность процессов, протекающих в межлопаточных каналах, по
нить это можно тем, что момент количества движения заштрихованной мас-
скоростям потока перед колесом и за ним. Это особенно удобно при прове-
сы
дении комплексных экспериментальных исследований интегральных харак-
11-2
и
1-21
1-11,
при вычитании сокращается (8.2).

теристик лопаточных машин.


d  mc x z  cz x   G   c x 11 2  z11 2  cz 11 2  x11 2 

 c x 2 21   z2 21   cz 221   x221  

 c 
 c 


Введя полярные координаты можно получить уравнение (8.4) в более удобной форме. При этом скорости раскладываются на радиальные
(индекс - r) и окружные (индекс - u) составляющие.
z
 c z 111   x111  
x 11  111 
1
x 11 2
  z1 2  cz1 2  x1 2  d
1
1
1

Окружные составляющие скорости потока лежат в координатной
плоскости X0Y.
8.2
30
M  G  c1u R1  c2u R2 
Моменты радиальных составляющих количества движения равны
8.8
нулю (рис. 8.2) так как радиальные составляющие скорости потока направлены параллельно оси Y.
где: М – сумма моментов всех сил, приложенных к какому либо объ-
Момент в точке 1 будет равен (рис. 8.5)
M1  G  c1u  R1
ему газа относительно начала координат (н а п р и м е р: момент на рабочем колесе турбины); с1u и c2u окружные составляющие скорости (н а п р и м
8.5
е р: скорость входа газа в рабочее колесо и выхода газа из рабочего колеса
соответственно).
Момент в точке 2 будет равен (8.6)
M2  G  c2u  R2
8.6
Z
c2r
Изменение момента силы, рассматриваемого участка потока газа (8.7)
M  M2  M1  G  c2u  R2  G  c1u  R1  G  c2u  R2  c1u  R1
8.7
c2u
При определении вращающего момента, действующего на рабочее
R2
R1
колесо лопаточной машины (н а п р и м е р турбины), теорема о моменте ко-
1
c1r
c1u
2
Y
0
личества движения может звучать следующим образом: изменение момента
количества движения потока газа относительно оси вращения рабочего ко-
X
леса в единицу времени равно сумме моментов всех внешних сил относительно той же оси.
Рассмотрим, какой момент создаст поток газа, протекающий в кана-
Рис. 8.2 Применение полярных координат к выводу уравнения Эйлера
ле (рис. 8.3). Конфигурация канала аналогична каналу проточной части рабочего колеса турбин радиального и осевого типов. Предположим, что рассматриваемые траектории движения частиц газа, движущегося по каналу –
В случае движения газа по инерции сумма моментов всех сил равна
плоские кривые, (движение плоскопараллельное) (рис. 8.3).
нулю и тогда получаем (8.9)
В данном случае суммарный момент всех сил, действующих на
c2u R2  c1u R1  const
стенки канала можно определить по формуле (8.8).
31
8.9
C1
1
C1
Z
9. Уравнение Бернулли
1
Закон Бернулли гласит – в стационарном потоке газа или жидкости
X
M
сумма статического и динамического давлений остается постоянной. Эта
u
сумма соответствует гидростатическому давлению покоящейся жидкости
Y=0
или газа.
C2
2
2
C2
Уравнение Бернулли относится к одной из форм записи уравнения
энергии, в которую не входит температура газа, а скорость движения связа-
R2
на с давлением и плотностью. Справедливо оно в случае одномерного об-
R1
ратимого потока при отсутствии внешних воздействий.
M
Для начала рассмотрим простейшую задачу гидростатики - дей-
R
Z=0
u
ствие, каких сил испытывает воздух (или любой другой газ), находящийся в
X
помещении с постоянной температурой и при отсутствии перемещений (рис.
9.1).
Y
z
б)
a)
Pàòì .
x
Рис. 8.3 Движение газа в плоском, криволинейном канале диска круглой
h
формы
а – к центру диска; б – вдоль оси диска.
P
Рис. 9.1 К выводу основного уравнения гидростатики
для выбранного столбика воздуха
Ратм. – атмосферное давление, которое испытывает столбик;
h – высота столбика; Р – уравновешивающее (гидростатическое) давление.
Специфика свойств газа в отличие от твердого тела состоит в том,
что давление в нем не зависит от ориентации площадки, на которой это
давление рассматривается. Каждый элемент газа испытывает одинаковое
32
боковое давление, уравновешиваемое давлением соседних элементов или
На практике наиболее часто встречаются процессы, при которых газ
давлением на стенках резервуара. Мысленно выделим вертикальный стол-
быстро движется, или тело перемещается в той же среде. В этих процессах
бик воздуха с поперечным сечением S. Этот столб будет испытывать атмо-
главную роль играют силы динамического происхождения.
сферное давление, обозначенное Ратм.. На площади S это давление вызо-
Основное уравнение гидростатики (9.2) содержит составляющую
вет силу (Ратм. S). В ту же сторону будет направлен и вес жидкого столба
(   g  z ), которая представляет потенциальную энергию единичного объе-
равный (ρghS), где ρ – плотность газа и g – ускорение свободного падения.
ма газа, P1 - представляет собой внутреннюю энергию газа. Следовательно,
Тот факт, что столб воздуха не проваливается вниз, свидетельствует о существовании давления в нижней части столба, которое и уравнове-
величины P1 и (   g  z ) можно интерпретировать, как энергию статического
шивает силы, направленные вниз. Это давление называют гидростатиче-
давления (напора). Так как справа от знака равенства уравнения (8.7) стоит
ским.
постоянная величина, уравнение (9.2) выражает собой закон сохранения
энергии для неподвижного газа.
Условие равновесия столба газа запишется так (9.1):
В том случае, если газ движется с некоторой постоянной скоростью
Pàòì .  S    g  h  S  P  S
с, единичный объем его будет обладать кинетической энергией
или
2
Рассмотрим движение газа в наклонной трубе переменного сечения
или
(рис. 9.2)
c2
c1
h1
давление.
h2
A2
A1
9.1
где: P0 - давление в резервуаре; P1 - полное уравновешивающее
рой хотят определить давление P0 . Отсчитывая эту координату от верхнего
P
среза столба, получаем отрицательную величину, которую обозначают z=-h.
Введя это обозначение в уравнение (9.1), получаем основное уравнение гидростатики (9.2).
Динамическое
давление
Статическое
давление
9.2
где; P1 - давление на уровне z (если точка с координатой z нахо-
Полное
давление
Обычно вместо высоты столбика вводят координату точки, в кото-
P1    g  z  P0
(ди-
намическое давление или динамический напор).
Pàòì .    g  h  P
P0    g  h  P1
  ñ2
l
Рис. 9.2 Течение газа в наклонной трубе переменного сечения
дится на дне резервуара, то P1 - давление на его дно).
33
Если обозначить согласно рис. 9.2: Р1 – статическое давление в
Изменение величины потенциальной энергии потока газа представ-
первом сечении; Р2 – статическое давление во втором сечении; с1 – ско-
ляется в виде (9.7)
рость потока газа в первом сечении; с2 – скорость потока газа во втором се-
Eï  m  g  h2  m  g  h1  m  g  h2  h1
чении; h1 – высота оси первого сечения относительно произвольно выбран-
9.7
ного уровня; h2 – высота оси второго сечения относительно уровня выбранного для первого сечения; А1 – площадь первого сечения; А2 – площадь вто-
Изменение величины кинетической энергии потока газа представ-
рого сечения.
ляется в
В связи с тем, что газ движется по наклонной трубе, в общем случае
виде (9.8)
необходимо учесть изменение энергии, связанное с гидростатической до-
Eê 
бавкой, возникающей из-за действия силы тяжести.
m  ñ2
2
2

m  ñ2
1
2


m 2
 ñ  ñ2
2
1
2

9.8
Работа, расходуемая на изменение скорости потока газа и его выПодставив уравнения (9.6, 9.7, 9.8) в уравнение (7.70) получим (9.9)
соты, равна сумме изменения потенциальной энергии и изменения кинетической энергии (9.3).
W=ΔЕп+ΔЕк
(P1  P2 )  V  m  g  h2  h1 
9.3

m 2
 ñ  ñ2
2
1
2

9.9
Разделив обе части уравнения (9.9) на элемент объема (ΔV) (это
Расходуемую работу можно определить как произведение силы на
можно сделать, так как объем остается постоянным вследствие принятого
перемещение (9.4)
допущения о том, что газ несжимаем при малых скоростях движения) и, учиW=FΔS
тывая, что плотность газа равна отношению массы к объему
9.4
Так как сила равна произведению давления на площадь ( P  A ), за-

пишем уравнение (9.4) в виде (9.5)
m
V
получим, раскрыв скобки (9.10)
W  P  A  S
9.5
P1  P2    g  h2    g  h1 
Заменив в уравнении 9.5 произведение площади (А) на перемещение (ΔS), объемом - получаем уравнение расходуемой работы в виде (9.6)
W  P  V  (P1  P2 )  V
  ñ2
2
2

  ñ2
1
2
9.10
Перенеся в одну сторону слагаемые с одинаковыми индексами,
обозначающими рассматриваемые сечения трубы, получим закон сохране-
9.6
ния энергии в следующем виде (9.11)
34
P1    g  z1 
  ñ12
2
 P2    g  z2 
  ñ22
2
 const
На практике для определения скорости газового потока используют
9.11
трубки Прандтля или трубки Вентури.
Полученное уравнение (9.11) является основным уравнением гид-
9.1. Трубка Прандтля – Пито
родинамики и называется уравнением Бернулли.
Закон Бернулли гласит – в стационарном потоке газа или жидкости
Трубка Прандтля - Пито (рис. 9.3) предназначена для определения
сумма статического и динамического давлений остается постоянной. Эта
разности полного и статического давлений с целью определения скоростей
сумма соответствует гидростатическому давлению (давлению торможения)
газовых потоков, а также относительную скорость объектов движущихся в
покоящегося газа (жидкости).
воздухе
В технической газовой динамике при малой плотности газа и малых
(н а п р и м е р самолеты) или в других газах.
перепадах высот пренебрегают составляющей (   g  z ), и применяют
уравнение Бернулли в таком виде (9.12)
9.12
2
1
Следует отметить, что закон сохранения энергии в таком виде
(9.11), точно выполняется только для газа или для жидкости полностью ли-
Рдин
  ñ2
  ñ2
P1  1 1  P2  2 2  P0*  const
2
2
шенных вязкости (идеальный газ или идеальная жидкость). Реальный газ
всегда обладает вязкостью, благодаря которой часть механической энергии
превращается в тепловую, нарушая тем самым справедливость уравнения
Бернулли. Однако, в том случае, если вязкость среды мала, например как у
воздуха при температурах умеренно отличающихся от нормальных условий
(t<300°C) и скоростях движения потока меньше 0,3 числа Маха, уравнение
Рис. 9.3 Трубка Прандтля
выполняется достаточно точно.
Кроме того, уравнение Бернулли, выражающее закон сохранения
Трубка Прандтля – Пито представляет собой цилиндрическую труб-
энергии, одинаково справедливо как в неподвижной системе координат, так
ку, ось которой устанавливается вдоль потока, на встречу ему. Через отвер-
и в любой равномерно движущейся системе.
стие 1 в этой трубки измеряется полное давление, которое реализуется при
Наиболее широко применяемые приборы для измерения скорости
торможении потока до нулевой скорости. Другое отверстие 2 располагается
движения потока газа также основаны на использовании связи между дав-
на боковой поверхности и служит для измерения статического давления.
лением и скоростью, которая следует из уравнения Бернулли.
35
Зная Päèí , которое равно
  ñ2
2
Для определения скорости потока воспользуемся уравнением Бери определив плотность газа,
нулли (9.11), так как ось канала, в котором движется газ, расположена горизонтально, оно будет выглядеть, как (9.12). Разность статических давлений
можно вычислить скорость потока по следующей формуле:
будет (9.13)
c
2  Päèí

  ñ2   ñ2 1
P  P2  P1  2 2  1 1   2  ñ22  1  ñ12
2
2
2


9.13
Воспользовавшись уравнением неразрывности (5.6), выразим ско-
9.2. Трубка Вентури
рость в узкой части трубы (9.14)
Трубка Вентури (рис. 9.4) устройство для замера расхода или ско-
cF
 F
c2  1 1 1  1  1  c1
 2F2
 2 F2
рости газа в трубопроводах. Представляет собой сужение на трубопроводе,
где скорость потока возрастает, а статическое давление соответственно
уменьшается.
9.14
Подставив уравнение (9.14) в (9.13), после ряда преобразований
получим
P 
Р2
2
2


  F 
1
 2   1    1   c12  1  ñ12 

2
  2   F2 


или
Р
  ñ2  
P  1 1   2
 1
2

Р1
2
2

 1   F1 








1
  

  2   F2 

В итоге получаем уравнение (9.13) в виде (9.15)
  ñ2  
P  1 1   1
 2
2
Рис. 9.4 Трубка Вентури

2

 F1 


    1

 F2 

Трубка Вентури (рис. 9.4) позволяет измерить разность статических
давлений в различных сечениях потока. По известной разности давлений
Из уравнения (9.15) можно найти скорость потока (9.16)
определяется скорость потока.
36
9.15
c12 
Можно ожидать, что шарик не удержится в струе воздуха, а отлетит
2  P

1
 2
1  


F 
  1   1

 F2 

2
9.16
в бок и упадет. Однако он располагается в струе и не может опуститься
вниз, так как струя его подпирает. Не поднимается он и вверх, так как его тянет вниз сила тяжести. Так и висит шарик в воздухе, вращаясь вокруг своей
оси (можно даже раскачивать струю в довольно широких пределах – шарик
Если необходимо найти скорость газа в узкой части трубы, то по-
как привязанный будет следовать за струей).
ступаем аналогично, только через уравнение (5.6) выражаем не c2 как в
Почему это происходит? Рассмотрим картину течения струи при
(9.14), а c1 .
возможном смещении шарика в сторону (рис. 9.5).
Если труба имеет круглое сечение и скорость потока газа не высо-
Шарик стесняет поток воздуха со своей левой стороны и вызывает
кая (то есть явление сжимаемости не проявляется и можно принять плот-
увеличение скорости струи. Увеличение скорости приводит к уменьшению
ность величиной неизменяемой) разность статических давлений равна (9.17)
давления в том месте по сравнению с другими участками, где давление рав-
 d  4 
1
P1  P2     c12  1   1
 d 2 

2


но атмосферному. Благодаря этому появляется сила Р, которая не дает шарику оторваться от воздушного потока.
9.17
Р
где: ρ – плотность газа; Р – статическое давление; с – скорость потока;
d – диаметр трубы; индексы 1 и 2 соответствуют участкам трубопровода.
При заданных размерах трубопровода и измеренной при помощи
дифференциального манометра разности давлений из уравнения (9.17)
можно определить среднюю скорость потока, следовательно, и расход газа.
Рис. 9.5 Шарик в воздухе
С помощью уравнения Бернулли можно дать объяснения ряду любопытных явлений, описанных ниже.
9.4. Эффект Магнуса.
9.3. Шарик висит в воздухе.
Этот эффект можно наблюдать на простом опыте. Если взять цилиндр, выполненный из плотной бумаги и скатить его по наклонной поверх-
Представим себе струю воздуха, направленную вертикально вверх
ности, расположенной на столе, то цилиндр при падении значительно от-
на которую положим шарик от пинг-понга (скорость струи не должна быть
клонится от своей прежней траектории движения по наклонной поверхности
большой, чтоб шарик не улетел далеко вверх).
37
и совершит полет в направлении противоположном ожидаемому направле-
значениям при величине окружной скорости, не превышающей четырех ско-
нию (рис. 9.6).
ростей набегающего потока.
Сравним подъемную силу, создаваемую обычным крылом и цилин-
Наклонная плоскость
дром вращающимся с окружной скоростью равной четырем скоростям набе-
Траектория
полета
цилиндра
Стол
Р
гающего потока, т.е. примем cö  4  c (экспериментально доказано, что в
этом предельном случае, и при меньшей окружной скорости, правомочно
Ожидаемая
траектория
полета
цилиндра
применение формулы (9.17)).
Подъемную силу крыла можно определить по формуле (9.18)
Pêð 
Рис. 9.6 Эффект Магнуса
Почему это происходит? Левая сторона цилиндра вращается в
(для современных крыльев он не превосходит 1,2).
ет его скорость. Правая сторона цилиндра вращается навстречу основному
потоку, и в этом месте скорость течения уменьшается. Согласно уравнению
Для сравнения выберем цилиндр той же площади диаметрального
Бернулли, давление слева будет меньше чем давление справа. Это и при-
сечения ( S  d  l ). Разделив (9.17), приняв cö  4  c , на (9.18) получим
ведет к появлению силы Р, которая обусловливает эффект Магнуса (то же
(9.19).
самое происходит во время подачи "крученых" мячей при игре в футбол, во-
1
   d  c  cö    l
P
  d  4  c  l
4 
2



 10
1
 1,2
Pêð
k ó  c  d  l
 kó    c 2  S
2
лейбол, теннис и т.п.).
Величина этой силы определяется формулой (9.17).
1
   d  c  cö    l
2
9.18
где: S - площадь крыла; k ó - экспериментальный коэффициент
сторону движения воздуха относительно цилиндра и тем самым увеличива-
P
1
 kó    c 2  S
2
9.19
9.17
Согласно 9.19 видно, что вращающийся цилиндр при равных условиях создает силу примерно в 10 раз большую, чем крыло.
где: d - диаметр цилиндра; l - его длина; c - скорость набегающе-
Попытка использовать описанное преимущество ротора была сде-
го потока; c ö - окружная скорость поверхности цилиндра;  - плотность
лана А. Флеттнером (1885-1961), который предложил использовать враща-
среды.
ющиеся цилиндры вместо парусов. Исследования, проведенные им, показали, что возникающая движущая сила была во много раз больше, чем давлеФормула (9.17) получена без учета действия сил вязкости среды.
ние ветра на неподвижный ротор. Выяснилось - используемая энергия ветра
Однако, экспериментальные данные оказываются близкими к вычисленным
38
примерно в 50 раз превышала ту, что затрачивалась на вращение ротора, и
зависела от частоты его вращения и скорости ветра. Кроме того, появилась
возможность плавания роторного судна против ветра переменными курсами
(галсами), близкими к линии ветра. Другими словами, для такого судна оставались действительными те законы плавания, которыми пользовались
обычные парусники. Но при этом его перспективы оценивались гораздо выше, поскольку площадь ротора по отношению к площади парусов обычного
парусника, сравнимого по водоизмещению с роторным судном, составляла
лишь 0,1-0,15 процента, а его (ротора) масса была примерно в 5 раз меньше, чем суммарная масса парусного вооружения.
В качестве первого опытного судна для его испытания использовали трехмачтовую шхуну "Букау" водоизмещением 980 т. (рис. 9.7). В 1924 году на ней вместо трех мачт поставили два ротора-цилиндра высотой 18,5 м
и диаметром 2,8 м. Их приводили в движение два электромотора постоянного тока, работавших от дизель-генератора мощностью 33 кВт (45 л.с.).
Общая площадь их диаметральных сечений была в 10 раз меньше
площади снятых с корабля парусов.
Являясь, по существу, парусниками, роторные суда обладали перед
ними преимуществами: всего один человек управлял движением роторов
при помощи рукояток. В бейдевинд (против ветра) эти суда шли до 30 градусов, тогда как у большинства обычных парусников угол между направлением
ветра и направлением движения составлял не менее 40-50 градусов. Скорость хода регулировалась скоростью вращения роторов, а маневрирование
- изменением направления их вращения. Роторные суда могли даже давать
задний ход.
Однако сложность конструкции роторных движителей, а главное - то
обстоятельство, что оснащенные ими суда продолжали оставаться парусниками со всеми недостатками, первый из которых - полная зависимость от
ветра, не привели к их широкому распространению, тем более, что в то время появились теплоходы, движение которых в меньшей степени зависит от
капризов природы.
Рис. 9.7 Судно с роторами Флеттнера
39
10. Теоретические параметры газа при его движении
Учитывая (10.4) из уравнения (10.5) можно найти изменение энтропии при протекании процесса движения газа (10.6)
В предыдущих разделах были рассмотрены следующие законы со-
dh 
хранения, для удобства запишем их еще раз:
dS 
• - закон сохранения количества движения (10.1)
c  dc 
1

 dP  0
10.1
dP

T
0
10.6
Из курса математики известно, что если dS  0 то S  const , следовательно, процесс происходит при неизменном значении энтропии и в
связи с этим называется - изоэнтропийным процессом.
В представленном виде уравнение справедливо для идеального га-
Прологарифмировав уравнение состояния P=ρRT, получим (10.7)
за (отсутствует трение).
ln P  ln   ln R  lnT
• - закон сохранения энергии (10.2)
h
c2
 const
2
10.2
Дифференцируем уравнение (10.7) и получаем (10.8)
dP d
dT

0
P

T
Продифференцировав это уравнение, получим (10.3)
dh  c  dc  0
10.7
10.8
10.3
Определим энтальпию по уравнению (4.20) с учетом (4.21), получим
Уравнение (10.3) справедливо и при движении газа с учетом трения,
(10.9)
главное чтобы не было теплообмена (процесс адиабатный).
h
Рассмотрев уравнения (10.1) и (10.3) как систему уравнений, можно
k
 R T
k 1
10.9
сделать вывод о том, что эта система уравнений применима для невязкого,
Подставив уравнение (10.9) в уравнение (10.6), получим (10.10)
нетеплопроводного газа. Решив ее, получим (10.4)
dh 
dP

0
dS 
10.4
1 k
dP 


 R  dT 
T  k 1
 
10.10
или, раскрыв скобки (10.11)
Из курса термодинамики известно уравнение (10.5)
T  dS  dh 
dP

dS 
10.5
40
k
dT
dP
R 

k 1
T
 T
10.11
Из уравнения (10.8) получим (10.12)
d 
 dP
dP
d
  P  k     P   k   const


dT dP d


T
P

10.12
d
dP
 P  k     const
Запишем уравнения состояния в виде (10.13)
 T 
P
R
 P
ln P  k  ln   ln P  ln  k  ln
 k

10.13

P
ln 1k
 1

Подставив уравнения (10.12) и (10.13) в уравнение (10.11), получим (10.14)
dS 
 dP d  dP
k

 R  

R
k 1
  P
 P

В связи с тем, что рассматриваемый процесс – изоэнтропийный
(S=0), сократив R (т.к. газовая постоянная не может быть равной нулю), рас-
k

P1
1
k

P2
 k2
 const
10.17
или (10.18)
крыв скобки и сделав элементарные преобразования, уравнение (10.14) за-
P1  1 


P2   2 
пишем в следующем виде (10.15)
1 dP
k d

0
k 1 P k 1 


  const


В итоге получим уравнение (10.17)
10.14
P
k
10.18
или (10.19)
10.15
1
1  P1  k
 
 2  P2 
 1 
 , получим (10.16)
 k  1
Разделив обе части уравнения (10.15) на 
dP
d
k
0
P




 P2

ln

 k

 2



  const


10.19
Воспользовавшись уравнением состояния P=ρRT, уравнение (10.18)
10.16
можно записать как (10.20)
 
1  R  T1
 T
 1 1  1
 2  R  T2  2  T2  2 
Интегрируем уравнение (10.16)
41
k
T1  1 


T2   2 
11 Примеры решения задач
k 1
10.20
11.1 Определение изменения энтропии
или (10.21)
1
Д а н о : масса воздуха 15 кг; начальная температура 25°С; темпе-
1  T1  k 1
 
 2  T2 
10.21
ратура после нагревания 50°С; давление атмосферное.
О п р е д е л и т ь изменение энтропии этого количества воздуха
Таким же образом, выразив из уравнения состояния (10.13) плот-
при его нагревании.
ность ρ и подставив ее в уравнение (10.18), получим уравнение (10.22)
Р е ш е н и е.
k
P T 
P1  P1 R  T2 
   1 2 
 

P2  R  T1 P2 
 P2 T1 
k
Изобарная теплоемкость воздуха в интересующем нас интервале
 êÄæ 
1
1
 P1  k
 
 P2 
1
P 
T
 2   1 
T1  P2 
тать, что теплоемкость в данном интервале не зависит от температуры.
1 k
k
Приняв величину теплоемкости постоянной и равной среднему значению -
 êÄæ 
10.22
1,00455 
 , получаем ошибку - 0,075%, что вполне приемлемо для
 êã  K 
или (10.23)
инженерных расчетов. Поскольку теплосодержание воздуха определяется
k
P1  T1  k 1
 
P2
 êÄæ 
температур меняется от 1,0038 
 до 1,0053  êã  K  . Можно счи êã  K 


 P1  k P1 T2
  
P2 T1
 P2 
T 
 2
как (11.1)
dQ  GcpdT
10.23
11.1
Подставив dQ из уравнения (11.1) в (4.18) получаем, вынося G и cp
Уравнения (10.18, 10.19, 10.20, 10.21, 10.22, 10.23) описывают изоэнтропийный процесс. Так как условия постоянства энтропии было получено
за знак интеграла, (11.2)
из сопоставления уравнений сохранения движения и сохранения энергии
T
S2  S1  Gc p ln 2
T1
(10.1, 10.3), то одно из этих уравнений можно заменить любым из вышеукаили в итоге
занных уравнений (10.18, 10.19, 10.20, 10.21, 10.22, 10.23). Но это допускается только при сделанных ограничениях об отсутствии трения при движении газа и теплообмена.
42
11.2
S2  S1  15  1,00455 ln
И в итоге по уравнению 5.5 находим скорость потока.
50  273,15
 êÄæ 
 1,21 

25  273,15
 êã  K 
c
О т в е т : изменение энтропии равно 1,21
êÄæ
êã  K
G
0,2
ì 

 84  
  F 2,38  0,001
ñ
О т в е т : скорость потока воздуха равна 84
ì
.
ñ
11.2 Определение скорости газа с помощью
закона сохранения массы
11.3 Определение температуры и скорости газа с помощью
Д а н о : воздух движется в канале, площадь сечения которого равна 0,001
м2
закона сохранения энергии
, полное давление потока 0,2 МПа; температура воздуха, заме-
ренная термометром 20°С; расход воздуха 0,2
Задача №1
êã
.
ñ
Д а н о : тело летит в воздухе со скоростью 1000
ì
.
ñ
О п р е д е л и т ь скорость движения воздуха в канале.
О п р е д е л и т ь, на какую температуру нагреется тело.
Р е ш е н и е.
Р е ш е н и е.
Для того, чтобы воспользоваться уравнением 5.5 необходимо знать
плотность газа, определим ее по уравнению состояния (11.3)
При полете с такой скоростью на поверхности тела образуется пограничный слой. В нем относительная скорость становится равной нулю, то
P    R T
11.3
 êã 
есть воздух как бы прилипает к поверхности, из-за чего температура становится равной температуре торможения.
 Äæ 
где: ρ - плотность газа 
 ; R - газовая постоянная для воздуха  êã  K  ;


ì 3 
Воспользовавшись формулой 6.32 получим уравнение (11.4) по ко-
 
торому можно определить разницу между температурой неподвижного воз-
Т - абсолютная температура Ê .
духа и температурой, которую передает пограничный слой поверхности тела.
Следовательно
6
 êã 
P
0,2  10


 2,38 

R  T 287  (20  273,15 )
ì 3 
T * T 
43
k 1
1,4  1
 c2 
 1000 2  497,7
2k R
2  1,4  287
11.4
О т в е т: тело нагреется примерно на 497,7°.
По таблице свойств газов определим, что коэффициент изоэнтропы
воздуха равен 1,4, а его газовая постоянная 287
Знак приблизительного равенства (  ) здесь стоит из-за того, что
истинная температура поверхности, обтекаемой газом отличается от темпе-
Äæ
.
êã  K
Воспользовавшись уравнением (6.35), получим величину скорости
ратуры торможения за счет теплового излучения в пространство и за счет
имеющегося "проскальзывания" самого пограничного слоя относительно по-
c
верхности тела. Для определения истинной температуры используют поправочный коэффициент (φ), который определяется экспериментальным путем


2k
2  1,4
ì 
R T * T 
 287  473,15  293,15   601,3  
k 1
1,4  1
ñ
(11.5).
О т в е т: теоретическая скорость потока на выходе из канала 601,3
k 1 1
c2
Tïîâ  Ò  

 c2  T 
k 2R
2  cp
11.5
ì
.
ñ
11.4 Определение необходимого противодавления с
Задача №2
помощью уравнения количества движения
Д а н о : температура торможения воздуха, движущегося в канале
Д а н о : температура воздуха в баллоне - 300 К; давление в бал-
200°С; его статическая температура 20°С.
лоне 0,15*106 Па; коэффициент изоэнтропы 1,4; газовая постоянная 287
Äæ
.
êã  K
О п р е д е л и т ь теоретическую скорость движения воздуха на
выходе из канала.
О п р е д е л и т ь, какое должно быть противодавление, чтобы при
открытии клапана теоретическая скорость потока равнялась 25 м/с.
Р е ш е н и е.
Р е ш е н и е.
Определим, чему равна температура по шкале Кельвина (по формуле 11.6), так как в исходных данных ее задали по шкале Цельсия.
Воспользуемся уравнением (7.14), из которого видно, что статическое давление на выходе равно (11.7):

T  t C  273,15
11.6
T *  200  273,15  473,15K
 k 1 c2
P1  P0*  1 
 1t

2

k
R  T0*

T  20  273,15  293,15K
44
k
 k 1



11.7
Р е ш е н и е.
1,4
 1,4  1
25 2  1,4 1
P1  0,15  10 6  1 

 0,149  10 6 Па


2

1
,
4
287

300


Обозначим: d=0,1 м; D=0,2 м; h=0,2 м; Pa=105 Па; tа=20°С.
1. Найдем температуру воздуха в абсолютных единицах
О т в е т: давление на выходе из баллона должно быть равным
0,149  106 Па.
Та = 273,15 + 20°С = 293,15 К
2. Переведем перепад давлений из мм.вод.ст. в Па (11.8)
11.5 Течение газа в канале
P 
h
0,2
 10 3 
 10 3  1960,8 Па
0,102
0,102
11.8
Д а н о : вентилятор всасывает воздух из атмосферы в помещение
через кольцевую щель (рис. 3.1) с внутренним диаметром 100 мм и внешним
3. Плотность атмосферного воздуха
диаметром 200 мм. Статическое давление воздуха в трубе измеряется с по-
a 
мощью водяного дифференциального манометра, разность уровней воды
равно 200 мм. Атмосферное давление 105 Па, температура воздуха 20°С.
Pa
êã
10 5

 1,189
R  Ta 287  293,15
ì 3
 Äæ 
Течение считать одномерным, потерями пренебречь, при данных
где: R = 287 
 – газовая постоянная воздуха, ищется по таб êã  Ê 
условиях воздух считать несжимаемым газом.
лицам свойств газов.
Н а й т и : массовый расход воздуха.
4. Условия задачи позволяют воспользоваться уравнением (9.11)
Внутренний диаметр
Бернулли (11.9)
P1  1  g  z1 
1  ñ21
2
 P2   2  g  z2 
 2  ñ22
11.9
2
или
P  P1  P2   2  g  z2 
Внешний диаметр
 2  ñ22
2
 1  g  z1 
1  ñ21
2
В связи с тем, что воздух при данных условиях считается газом не-
Рис. 11.1 Кольцевая щель
сжимаемым то
45
2  1   , воздух движется горизонтально, поэтому
виде (11.10)
T0*
P 
  ñ2
1
3
4
P1
c1 ;G1
d1
P0*
Учитывая это, последнее уравнение можно записать в следующем
2
2
c2 ;G2
d3
1
ì
цию неподвижным c1  0
.
ñ
d2
z2  z1 . Кроме того, считаем атмосферный воздух перед входом в вентиля-
2 P2
1
11.10
2
3
c3 ;G3
P3
3
T11 ;G11
и из него найти скорость воздуха в воздуховоде
Рис. 11.2 Воздушный эжектор
P
ì
1960,8
c  2
 2
 57,4
ñ

1,189
1 – сопло; 2 – камера подвода; 3 – смесительная камера; 4 – диффузор.
5. Площадь проходного сечения воздуховода
F

4

2
 D d
2
  4  0,2

2
2

Эжектор состоит из круглого сопла (1), из которого вытекает активная струя воздуха, камеры подвода (2) отсасываемого из помещения возду-
 0,1  23,56  10
3
ха, смесительной камеры (3), в конце которой поток выравнивается и диф-
м2
фузора (4).
6. Массовый расход воздуха найдем по уравнению неразрывности (5.5)
G    F  c  1,189  23,56  10 3  57,4  1,61
О т в е т: расход воздуха 1,61
Д а н о: давление торможения перед соплом P0*  0,18 МПа; температура торможения перед соплом T0*  400 К; диаметр сопла 200 мм;
êã
ñ
давление в камере подвода отсасываемого воздуха P1  0,1 МПа; темпера-
êã
ñ
тура отсасываемого воздуха T01  300 К; диаметр смесительной камеры
d2  60 мм; диаметр выходного сечения диффузора d3  80 мм.
Н а й т и массовый расход через сопло эжектирующего воздуха
11.6 Воздушный эжектор
( G1 ), массовый расход эжектируемого воздуха ( G11 ) и параметры потока за
Рассчитать эжектор (рис. 3.2).
46
В связи с тем, что скорость отсасываемого воздуха в сечении 1-1
диффузором, такие как давление, температуру и скорость ( P3 ;T3 ; c3 ). Ко-
1
камеры смешения в проекции на ось эжектора равна нулю ( c 1  0 ), то по-
G1
эффициент эжекции задан 1  0,3 .
G1
следнее уравнение можно переписать так (11.16)
Р е ш е н и е.


c 21 
c 22


1
1
G1   c p  T1 
 G  c p  T1  G2   c p  T2 
1
2 
2





1. Запишем уравнение сохранения движения в проекции на ось





11.16
эжектора между контрольными сечениями 1-1 и 2-2 камеры смешения
4. Вспомним, что уравнение состояния совершенного газа (11.17)
(11.11)
G2  c2  G1  c1  F2  P2  P1
11.11
P=ρRT
11.17
Воспользовавшись уравнением сохранения массы 5.5, запишем
По этому уравнению (11.17) найдем плотность газа на входе в соп-
уравнение 11.11 в следующем виде (11.12)
 2  F2  c 2  1  F1  c 2  F2  P2  P1
2
1
ло по заторможенным параметрам
11.12
0* 
2. Уравнение сохранения массы между контрольными сечениями 1и
1
2-2 камеры смешения (11.13)
P0*
R  T0*

êã
0,18  10 6
 1,568
287  400
ì 3
5. По справочнику свойств газов найдем следующие значения и
1
G1  G  G2
11.13
1
примем их постоянными: теплоемкость воздуха сР=1010
3. Используя уравнение энергии Е=сvT, уравнение сохранения энергии между контрольными сечениями 1-1 и 2-2 камеры смешения с учетом то-
ент изоэнтропы k=1,4; газовая постоянная R=287
го, что уравнение энергии потока единичной массы 6.24 имеет вид (11.14)
T* T 
c2
 const
2  cp
Äæ
; коэффициêã  Ê
Äæ
.
êã  Ê
6. Воспользуемся уравнениями изоэнтропийного процесса (10.18,
11.14
10.19, 10.20, 10.21, 10.22, 10.23) для нахождения неизвестных величин
можно записать (считая сv величиной постоянной) (11.15)
 
2



c 21 
c11 
c 22



1
1
G1   c p  T1 
 G   c p  T1 
  G2   c p  T2 
1
2 
2 
2









P
T1  T0*   1
 P*
 0


 11.15


47
k 1
1,4 1
 k
 0,1 10 6  1,4


 400  
 338,16 К
 0,18  10 6 




1
1
 P k
 0,1 10 6  1,4
  1,03 êã
1  0*   1*   1,568  
 0,18  10 6 
P 
ì 3


 0
11. Подставим найденные численные величины в уравнение сохранения массы (пункт 2, уравнение 11.13), после проведения элементарных
преобразований, получим (11.19)
G1  G1  G2   2  F2  c2
7. Скорость газа на выходе из сопла
1
c1 


0,114  0,0342   2 
ì
2k
2  1,4
 R  T0*  T1 
 287  400  338,16   352,5
ñ
k 1
1,4  1
G1  1  F1  c1  1,03 
4

 20  10



c 21 
c 22


1
1
G1   c p  T1 
 G  c p  T1  G2   c p  T2 
1
2 
2





êã
ñ

c 22

 0,114  0,0342   1010  T2 
2


2 


4

4

 60  10

 60  10  3
  c  1,03  4  20  10   352,5
  P  0,1 10 
3 2

2
3 2
2
2
T2  376,6 
2

c 22
2020
(11.20)
13. Запишем уравнение состояния совершенного газа (11.21)
P2  2  R  T2  287  2  T2
6
2





в итоге
 2  F2  c  1  F1  c  F2  P2  P1
1






352,5 2 
0,114  1010  338,16 
 0,0342  1010  300 

2 

ных преобразований, получим (11.18)
2
(11.19)
преобразований, получим (11.20)
чества движения (пункт 1, уравнение 11.12), после проведения элементар-
2
2
нения энергии (пункт 3, уравнение 11.16), после проведения элементарных
10. Подставим найденные численные величины в уравнение коли-
2
2
12. Подставим найденные численные величины в уравнение сохра-
êã
 352,5  0,114
ñ
9. Массовый расход отсасываемого воздуха
G11  0,3  G1  0,3  0,114  0,0342
 c
2  c2  52,42
разрывности (5.5)
3 2
4

 60  10 3
в итоге
8. Массовый расход воздуха через сопло ищем по уравнению не-


в итоге
(11.21)
14. Решаем аналитически систему из четырех уравнений (11.18,
 2  c  P2  1,142  10
2
2
5
11.19, 11.20 и 11.21)
(11.18)
48
18. По уравнению состояния совершенного газа (пункт 13, уравне-
 2  c 2  P2  1,142  10 5
2
ние 11.21) находим давление - P2
2  c2  52,42
T2  376,6 
P2  287  1,0355  375,3  0,1115  10 6 Па
c 22
19. Число Маха на выходе из камеры смешения ищем для того, что-
2020
бы проверить имеем ли мы право, использовать допущение о том, что при
P2  287  2  T2
заданных параметрах воздух можно считать несжимаемым газом.
получаем, квадратное уравнение в виде (11.22)
c22
 2538 ,6  c2  126000  0
M2 
(11.22)
c2
k  R  T2
50,6

1,4  287  375,3
 0,13
Так как M 2 << 0,3 то параметры потока в выходном сечении можно
15. Находим скорость c2
c2 
найти, принимая плотность воздуха постоянной и равной плотности воздуха
в камере смешения ( 2  3 ).
2538 ,6  2538 ,62  4  1 126000
2 1
Один корень имеет значение c2  2488
20. Из уравнения неразрывности (5.5) найдем скорость на выходе из
ì
ì
, другой c2  50,6
.
ñ
ñ
диффузора с3.
G   2  F2  c2   2  F3  c3
Скорость воздуха на выходе из сопла c1 , не может быть меньше
скорости воздуха в смесительной камере c2 . Поэтому в качестве действительной скорости следует выбрать меньшую, то есть c2  50,6
c3 
ì
.
ñ
ì
F2  c2   d 22
d2
4
0,06 2


 c2  2  c2 
 50,6  28,46
2
2
2
ñ
F3
4
  d3
d3
0,08
21. По уравнению Бернулли 9.11 давление на выходе из диффузора
Р3 (плотность воздуха в смесительной камере и на выходе из диффузора
16. Подставляя значения скорости, в уравнение пункта 12 (11.20)
считаем одинаковой).
находим температуру - T2
T2  376,6 
P2   2  g  z 
50,62
 375,3 К
2020
2
 P3   2  g  z 
 2  ñ23
2
Сократив  2  g  z , находим Р3.
17. Подставляя значения скорости, в уравнение пункта 11 (11.19)
находим плотность -  2
2 
 2  ñ22
P2 
êã
52,4
 1,0355
50,6
ì 3
 2  ñ22
2


 2  ñ23
2

 P3


1,0355
P3  P2  2  ñ22  ñ23  0,1115  10 6 
 50,62  28,46 2
2
2
49

13. Как звучит закон сохранения энергии?
P3  0,112  10 6 Па
14. Написать уравнение состояния идеального газа.
22. Температуру воздуха на выходе из диффузора Т3 определим по
15. Что является параметрами торможения?
уравнению состояния совершенного газа (пункт 4, уравнение 11.17)
16. Как
0,112  10 6
T3 
 377 К
1,0355  287
выглядит
уравнение
сохранения
энергии
при
определении
сохранения
энергии
при
определении
сохранения
энергии
при
определении
энтальпии торможения?
17. Как
выглядит
уравнение
температуры торможения?
О т в е т : расход эжектирующего воздуха 0,114
18. Как
êã
; расход отсаñ
выглядит
уравнение
скорости потока газа?
êã
сываемого воздуха 0,0342
; давление за диффузором 0,112  106 Па;
ñ
температура воздуха на выходе из диффузора 377 К; скорость потока на
ì
выходе из диффузора 28,46
.
ñ
19. Как изменится температура газа при прохождении его через проточную
часть турбины, работающую на генератор или другой потребитель
энергии?
20. Как
изменится
температура
газа
при
прохождении
его
через
проточную часть турбины, рабочее колесо которой не вращается?
Вопросы для самопроверки
21. На использовании, каких свойств основана работа турбодетандеров?
22. В каком случае энтальпия газа достигает своего максимального
1. Перечислить законы сохранения.
значения?
2. Что представляет собой изоэнтропийный процесс?
3. Чем
отличается
универсальная
газовая
23. С помощью какого прибора определяется температура торможения?
постоянная
от
газовой
24. Как узнать величину статической температуры?
постоянной конкретного газа?
25. Как звучит закон сохранения импульса?
4. Как звучит закон сохранения массы?
26. Какие силы относят к внутренним силам?
5. Напишите формулу закона сохранения массы.
27. Какие силы относят к внешним силам?
6. Как
изменяется
скорость
потока
газа
при
движении
в
канале
скорость
потока
газа
при
движении
в
канале
28. Как выглядит уравнение сохранения импульса при определении скоро-
диффузорной формы?
7. Как
изменяется
сти потока газа?
29. Как звучит закон сохранения момента импульса?
конфузорной формы?
30. Напишите уравнение для определения момента сил потока газа, дей-
8. Что такое энергия?
ствующего на стенки канала.
9. Назовите формы энергии.
31. Как звучит закон Бернулли?
10. Что такое кинетическая энергия?
32. Какой из законов сохранения выражает уравнение Бернулли?
11. Что такое потенциальная энергия?
33. При каких условиях справедливо уравнение Бернулли?
12. Что такое внутренняя энергия?
34. Напишите уравнение Бернулли.
50
ЗАДАЧИ ДЛЯ САМОСТОЯТЕЛЬНОГО РЕШЕНИЯ
35. Чем отличается полное давление газа от статического давления?
36. Каким прибором измеряют полное давление газа?
ЗАДАЧА 1
37. Каким образом определяют статическое давление газа?
38. Что представляет собой трубка Прандтля?
Рассчитать параметры сопла
c 0 - скорость потока на входе в сопло – м/с;
Дано:
39. Для чего предназначена трубка Прандтля?
40. Что представляет собой трубка Вентури?
P0 - статическое давление на входе – МПа;
41. Для чего предназначена трубка Вентури?
T * - температура торможения на входе – К;
P1 - статическое давление на выходе – Па;
k - показатель изоэнтропы – 1,4;
R - газовая постоянная – 287 Дж/(кг К);
G - массовый расход газа – кг/с.
42. Приведите пример эффекта Магнуса.
43. Что представляет собой парус Флаттнера?
Найти:
давление торможения на входе; статическую температуру (теоретическую) и скорость (теоретическую) на выходе; площадь и
диаметр сопла в выходном сечении.
№ вар
c0
P0
T*
P1
G
1
2
3
4
5
6
7
8
9
10
100
110
95
98
112
109
99
104
115
112
1,5
1,6
1,45
1,4
1,55
1,65
1,38
1,42
1,39
1,58
527,625
532,125
525,205
527,120
528,450
527,360
526,455
529,545
528,950
526,780
1098330
1106980
1086854
1186472
1097548
1096542
1089548
1083654
1081265
1099898
0,5
0,6
0,55
0,65
0,48
0,49
0,59
0,52
0,51
0,62
ЗАДАЧА 2
Рассчитать параметры сопла
Дано:
51
M - число Маха на выходе из сопла (по теор. параметрам) ;
P * - давление торможения на входе – МПа;
T * - температура торможения на входе – С;
k - показатель изоэнтропы – 1,4;
R - газовая постоянная – 287 Дж/(кг К);
d - диаметр критического сечения – мм.
Найти:
6
7
8
9
10
теоретическую скорость на выходе из сопла; площадь и диаметр
сопла в выходном сечении, массовый расход газа.
2010450
2006005
2045700
2540000
2000055
0,812
0,786
0,736
0,869
0,833
450
459
457
452
453
22
24
21
19
23
ЗАДАЧА 4
№ вар
M
P*
T*
d
1
2
3
4
5
6
7
8
9
10
1,8078
1,7980
1,6895
1,8540
1,8975
1,7654
1,7124
1,8065
1,6984
1,9345
3
3,25
3,02
2,98
3,32
3,38
3,15
2,95
2,84
3,64
500
505
495
498
503
499
501
496
497
502
15,5
15,1
14,5
16
15
16,5
17
13
14
20
Рассчитать параметры сопла
M - число Маха на выходе из сопла ;
P0 - статическое давление на входе –МПа;
Дано:
T0 - статическая температура на входе –К;
c - скорость потока на входе –м/с;
k - показатель изоэнтропы – 1,4;
R - газовая постоянная – 287 Дж/(кг К);
 - коэффициент скорости сопла ;
ЗАДАЧА 3
Найти:
Рассчитать параметры сопла
Дано:
448
452
451
446
449
P * - давление торможения на входе в сопло –Па;
T0 - статическая температура на входе –К;
№ вар
P1- статическое давление на выходе –МПа;
d - диаметр выходного сечения – мм;
T * - температура торможения на входе –К;
k - показатель изоэнтропы – 1,4;
R - газовая постоянная – 287 Дж/(кг К);
Найти: статическое давление на входе в сопло; теоретический расход газа.
№ вар
P*
T0
P1
T*
d
1
2
3
4
5
2015400
2000985
2000545
1900500
1950060
443
458
452
460
455
0,808
0,824
0,834
0,798
0,792
450
460
465
470
458
20
21
20
19
18
давление и температуру торможения на входе; статическое
давление на выходе из сопла; потерю кинетической энергии
сопла.
M
P0
T0
c

1
1
1,3
800
80
0,98
2
1,1
1,35
795
84
1,12
3
1,4
1,29
798
79
1,02
4
1,2
1,34
804
69
0,97
5
1,25
1,37
803
74
0,99
6
1,15
1,41
809
85
1,06
7
1,45
1,36
790
75
1,15
8
1,32
1,31
799
78
1,03
9
1,24
1,28
807
86
1,02
10
1,18
1,26
797
73
1,14
ЗАДАЧА 5
Дано: для сопла с параметрами:
52
3
4
5
6
7
8
9
10
dкр - диаметр критического сечения сопла – мм;
dвых - диаметр выходного сечения сопла – мм;
P * - давление торможения на входе – МПа;
T * - температура торможения на входе – К;
k - показатель изоэнтропы – 1,4;
R - газовая постоянная – 287 Дж/(кг К);
Найти:
№ вар
1
2
3
4
5
6
7
8
9
10
число Маха на выходе по теоретическим параметрам; теоретический расход газа.
dкр
dвых
P*
T*
15
20
1
600
18
25
0,98
602
14
23
1,12
605
19
27
1,04
596
20
26
1,09
599
22
28
1,16
609
25
30
1,21
605
21
29
0,95
598
20
24
0,98
602
13
20
1,19
603
Дано:
T * - температура торможения –К;
k - показатель изоэнтропы – 1,4;
R - газовая постоянная – 287 Дж/(кг К);
c - скорость на входе –м/с.
Найти:
№ вар
1
2
3
4
5
6
7
8
9
10
500
502
давление торможения на входе; статическое давление на входе;
теоретическую скорость газа на выходе.
c
P1
H
T*
100,5
0,1
600
100
99,5
0,12
602
109
101,3
0,15
605
94
100,9
0,19
596
104
100,1
0,09
599
101
99,6
0,07
609
97
99,8
0,14
605
99
101,3
0,13
598
96
98,9
0,08
602
108
100,3
0,18
603
105
ЗАДАЧА 8
Рассчитать параметры сопла
T
3
3,25
для сопла с параметрами:
P1 - статическое давление на выходе - МПа;
теоретическую скорость на выходе из сопла; площадь и диаметр
сопла в выходном сечении, теоретический массовый расход газа.
*
dкр
M
P*
1,8078
1,7980
14
19
20
22
25
21
20
13
кДж/кг;
dкр - диаметр критического сечения –мм.
1
2
505
496
499
509
505
498
502
503
H - изоэнтропийный перепад энтальпий от полных параметров –
Дано: рассчитать параметры сопла
Дано: M - число Маха на выходе из сопла (по теор. параметрам) ;
P * - давление торможения на входе –МПа;
T * - температура торможения на входе – 0 С;
k - показатель изоэнтропы – 1,4;
R - газовая постоянная – 287 Дж/(кг К);
№ вар
3,02
2,98
3,32
3,38
3,15
2,95
2,84
3,64
ЗАДАЧА 7
ЗАДАЧА 6
Найти:
1,6895
1,8540
1,8975
1,7654
1,7124
1,8065
1,6984
1,9345
Дано:
15
18
53
P1 - статическое давление на выходе – Па;
P * - давление торможения на входе –МПа;
6
7
8
9
10
T * - температура торможения на входе – 0 С;
k - показатель изоэнтропы – 1,4;
R - газовая постоянная – 287 Дж/(кг К);
dкр - диаметр критического сечения – мм.
Найти:
№ вар
1
2
3
4
5
6
7
8
9
10
теоретическую скорость на выходе из сопла; площадь и диаметр
сопла в выходном сечении, теоретический массовый расход газа; число Маха на выходе из сопла (по теоретическим параметрам)
dкр
P1
P*
T*
5
5,20х10
3,05
400
10
5,24х105
3,29
406
13
5,05х105
3,12
401
9
5,16х105
3,07
398
15
5,32х105
3,20
394
20
5,28х105
3,24
409
19
5,21х105
3,13
407
11
5,12х105
3,11
395
24
5,03х105
3,01
403
16
5,17х105
3,04
399
8
k - показатель изоэнтропы – 1,4;
R - газовая постоянная – 287 Дж/(кг К);
T1 - теоретическая статическая температура на выходе – 0 С;
P1 - статическое давление на выходе – МПа.
Найти:
статические давление и температуру на выходе; теоретическую
скорость газа на выходе.
*

c
P*
605
601
612
611
603
600
595
599
602
590
давление торможения на входе; площадь и диаметр сопла в
выходном сечении, теоретический массовый расход газа.
№ вар
M
dкр
T1
P1
1
2
3
4
5
6
7
8
9
10
2,1073
2,1004
2,0965
2,0875
2,1234
2,214
2,1954
2,0347
2,0994
2,0961
8
9
7
12
16
20
13
12
10
19
20
22
18
21
25
19
28
33
21
24
0,21
0,24
0,19
0,18
0,23
0,17
0,25
0,22
0,16
0,15
ЗАДАЧА 11
T
1,65
1,66
1,61
1,54
1,56
0,97
0,95
0,93
1,04
1,07
Рассчитать параметры сопла
k - показатель изоэнтропы – 1,4;
R - газовая постоянная – 287 Дж/(кг К);
1
2
3
4
5
603
612
592
604
610
M - число Маха в выходном сечении;
dкр - диаметр критического сечения – мм;
Дано:
Дано: для сопла с параметрами:
P * - давление торможения на входе – МПа;
T * - температура торможения на входе – К;
c - скорость газа на выходе – м/с;
 - коэффициент скорости сопла ;
№ вар
600
598
597
591
606
ЗАДАЧА 10
ЗАДАЧА 9
Найти:
1,69
1,70
1,65
1,49
1,47
0,98
0,99
1,02
1,06
0,94
Дано: для сопла с параметрами:
P * - давление торможения на входе –МПа;
T * - температура торможения на входе –К;
54
dвых - диаметр выходного сечения –мм;
k - показатель изоэнтропы – 1,4;
R - газовая постоянная – 287 Дж/(кг К);
Найти:
№ вар
1
2
3
4
5
6
7
8
9
10
статические давление и температуру на выходе; теоретическую
скорость газа на выходе, число Маха в выходном сечении.
dкр
dвых
P*
T*
0,5
600
10
13
0,65
605
13
20
0,66
601
9
25
0,61
612
15
23
0,54
611
20
27
0,56
603
19
26
0,69
600
11
28
0,70
598
24
30
0,65
597
16
29
0,49
591
8
24
1
2
3
4
5
6
Найти:
№ вар
1
2
3
4
5
6
7
8
9
10
450
458
459
444
460
447
статические давление и температуру на выходе; скорость газа
на выходе, коэффициент потерь кинетической энергии сопла.
*


P*
T
2,02
1,98
2,12
2,04
2,09
2,16
2,21
1,95
1,98
2,19
400
394
409
407
395
403
399
406
401
398
99,7
0,98
0,97
1,04
1,07
0,99
1,02
1,06
0,94
0,97
ЗАДАЧА 14
Дано: для сопла с параметрами:
P * - давление торможения на входе – 0,5 МПа;
T * - температура торможения на входе – К;
T
605
600
598
597
591
600
0,99
1,02
1,06
0,94
k - показатель изоэнтропы – 1,4;
R - газовая постоянная – 287 Дж/(кг К);
теоретическую скорость газа на выходе; статические температуру и давление на выходе.
*

c
P*
1,6 x 106
1,9 x 106
1,2 x 106
1,4 x 106
1,8 x 106
1,1 x 106
446
453
457
451
Дано: для сопла с параметрами:
P * - давление торможения на входе –МПа;
T * - температура торможения на входе –0 С;
 - теоретический теплоперепад на сопло –кДж/кг;
 - коэффициент скорости сопла ;
Дано: для сопла с параметрами:
T * - температура торможения на входе –К;
P * - давление торможения на входе –Па;
k - показатель изоэнтропы – 1,4;
R - газовая постоянная – 287 Дж/(кг К);
c - скорость газа на выходе –м/с;
 - коэффициент скорости сопла.
№ вар
605
601
612
611
ЗАДАЧА 13
ЗАДАЧА 12
Найти:
2,1 x 106
1,9 x 106
1,6 x 106
1,5 x 106
7
8
9
10
dкр - диаметр критического сечения –мм;
0,98
0,97
0,95
0,93
1,04
1,07
dкр - диаметр критического сечения – 10 мм;
dвых - диаметр выходного сечения –мм;
k - показатель изоэнтропы – 1,4;
55
0,98
1,02
1,06
0,94
0,95
0,93
1,04
1,07
0,98
0,97
R - газовая постоянная – 287 Дж/(кг К);
Найти:
№ вар
1
2
3
4
5
6
7
8
9
10
6
7
8
9
10
число Маха на выходе, статические давление и температуру,
теоретическую скорость газа в выходном сечении.
dкр
dвых
P*
T*
0,5
800
8
13
0,65
805
9
20
0,66
801
7
25
0,61
812
12
23
0,54
811
16
27
0,56
803
20
26
0,69
800
13
28
0,70
798
12
30
0,65
797
10
29
0,49
791
19
24
T1 - статическая температура на выходе – К;
P1 - статическое давление на выходе –МПа.
k - показатель изоэнтропы – 1,4;
R - газовая постоянная – 287 Дж/(кг К);
Дано: для сопла с параметрами:
отношение давления торможения на входе к стат. давлению на
выходе;
T * - температура торможения на входе –К;
P * - давление торможения на входе –МПа;
k - показатель изоэнтропы – 1,4;
R - газовая постоянная – 287 Дж/(кг К);
Найти: число Маха и статическое давление, а так же теоретическую скорость
газа в выходном сечении.
№ вар
T*
P*
1
2
3
4
5
2,19
2,22
2,18
2,26
2,27
600
605
600
598
597
1,3
1,2
1
0,9
1,5
1,8
0,6
1,4
1,7
0,7
Дано: газ, вылетающий из сопла, имеет следующие параметры
H - изоэнтропийный перепад энтальпий от полных параметров – кДж/кг;
Найти:
P*
P1
591
600
605
601
612
ЗАДАЧА 16
ЗАДАЧА 15
P*
P1
2,15
2,13
2,27
2,29
2,13
Температуру торможения и давление торможения на входе в
сопло.
№ вар
H
P1
T1
1
2
3
4
5
6
7
8
9
10
5,05
5,14
4,98
5,18
5,20
4,97
4,99
5,11
5,14
5,07
0,1
0,3
0,4
0,15
0,7
0,09
0,8
0,6
0,75
0,25
300
305
301
312
311
303
300
298
297
291
ЗАДАЧА 17
Дано: для сопла с параметрами:
G - массовый расход газа –кг/с;
T * - температура торможения на входе –К;
P * - давление торможения на входе –МПа;
P1 - статическое давление на выходе –Па;
56
k - показатель изоэнтропы – 1,4;
R - газовая постоянная – 287 Дж/(кг К);
Найти:
№ вар
1
2
3
4
5
6
7
8
9
10
6
7
8
9
10
следующие теоретические параметры: статическую температуру и скорость газа в выходном сечении, площадь выходного сечения.
P1
G
P*
T*
0,3
429
0,45
123000
0,45
405
0,47
124000
0,55
401
0,41
127000
0,38
412
0,39
129000
0,39
411
0,49
115000
0,49
403
0,52
118000
0,32
400
0,47
129000
0,31
398
0,43
114000
0,32
397
0,41
128000
0,41
423
0,38
113000
0,59
0,52
0,51
0,62
0,41
S êð - площадь критического сечения – м2;
T êð - статическая температура в критическом сечении – К;
P * - давление торможения на входе – МПа;
T1 - статическая температура (теоретическая) в выходном сечении – К;
k - показатель изоэнтропы – 1,4;
R - газовая постоянная – 287 Дж/(кг К);
Дано: для сопла с параметрами:
G - массовый расход газа – кг/с;
T * - температура торможения на входе – К;
P * - давление торможения на входе – 0,25 МПа;
P1- статическое давление на выходе – Па;
k - показатель изоэнтропы – 1,4;
R - газовая постоянная – 287 Дж/(кг К);
1
2
3
4
5
108000
119000
104000
118000
103000
Дано: для сопла с параметрами:
ЗАДАЧА 18
№ вар
0,32
0,27
0,23
0,21
0,18
ЗАДАЧА 19
Найти:
Найти:
503
500
498
497
523
следующие теоретические параметры: скорость газа в выходном сечении, площадь выходного сечения; определить, какое
должно быть сопло - суживающееся или сверхзвуковое.
P1
G
P*
T*
0,3
529
0,25
113000
0,55
505
0,27
114000
0,65
501
0,21
117000
0,48
512
0,19
119000
0,49
511
0,29
105000
массовый расход газа; теоретическую скорость газа и статическое давление на выходе; диаметр выходного сечения.
№ вар
S êð
T êð
P*
T1
1
2
3
4
5
6
7
8
9
10
4х10-6
400
411
403
400
398
397
423
429
405
401
0,55
0,56
0,57
0,51
0,49
0,59
0,62
0,57
0,53
0,51
300
305
301
312
311
303
300
298
297
291
5х10-6
7х10-6
4х10-6
9х10-6
8х10-6
6х10-6
12х10-6
6х10-6
9х10-6
ЗАДАЧА 20
Дано: для сопла с параметрами:
Tкр - статическая температура газа в критическом сечении – 2270 С;
57
T1 -
статическая температура (теоретическая) на выходе из сопла –
Найти: массовый расход газа; теоретическую скорость газа на выходе.
1270 С;
dкр - площадь критического сечения – 5 мм2;
ЗАДАЧА 23
P * - давление торможения на входе – 0,5 МПа;
k - показатель изоэнтропы – 1,4;
R - газовая постоянная – 287 Дж/(кг К);
Найти:
Дано: для суживающегося сопла с параметрами:
T * - температура торможения на входе – 600 К;
P * - давление торможения на входе – 2 МПа;
P1 - статическое давление на выходе из сопла – 105 Па;
Sвых - площадь выходного сечения – 1000 мм2;
k - показатель изоэнтропы – 1,4;
R - газовая постоянная – 287 Дж/(кг К);
массовый расход газа; теоретическую скорость газа и статическое давление на выходе; площадь выходного сечения.
ЗАДАЧА 21
Найти: массовый расход газа; теоретическую скорость газа на выходе.
Дано: для суживающегося сопла с параметрами:
dвых - диаметр выходного сечения – 10-2 м;
ЗАДАЧА 24
T * - температура торможения на входе – 2000 С;
P * - давление торможения на входе – 10 МПа;
P1- статическое давление в выходном сечении –100000 Па;
k - показатель изоэнтропы – 1,4;
R - газовая постоянная – 287 Дж/(кг К);
Дано: для сопла Лаваля с параметрами:
P * - температура торможения на входе –350 К;
k - показатель изоэнтропы – 1,4;
R - газовая постоянная – 287 Дж/(кг К);
P1 - статическое давление на выходе из сопла – 0,1x106 Па;
Sвых - площадь выходного сечения – 1000 мм2;
P * - давление торможения на входе – 2x106 Па;
Найти: массовый расход газа; теоретическую скорость газа на выходе.
Найти: массовый расход газа; теоретическую скорость газа на выходе.
ЗАДАЧА 22
Дано: для сопла Лаваля с параметрами:
T * - температура торможения на входе – 500 К;
P1- статическое давление на выходе из сопла – 100000 Па;
Sвых - площадь выходного сечения – 100 мм2;
P * - давление торможения на входе – 0,20 МПа;
k - показатель изоэнтропы – 1,4;
R - газовая постоянная – 287 Дж/(кг К);
58
Течение газа в соплах и каналах
3. ПЕРЕЧЕНЬ ВОПРОСОВ ПО КУРСУ
"ТЕХНИЧЕСКАЯ ГАЗОВАЯ ДИНАМИКА"
27.
Адиабатный процесс расширения газа в соплах в диаграмме h-s.
28.
Вывод формулы теоретической скорости истечения газа из сопел.
29.
Действительное истечение газа из сопел. Коэффициент скорости сопла.
Потеря энергии в соплах.
30.
Вывод формулы теоретического расхода газа через сопло.
31.
Изменение расхода газа через сопло в зависимости от перепада давления. Понятие коэффициента расхода сопла.
32.
Относительный расход газа через сопло. Удельный и приведенный
расход.
33.
Расход несжимаемого газа или жидкости через канал произвольного
сечения. Связь между скоростью и давлением в потоке газа.
34.
Влияние давления перед и за соплом на расход газа.
35.
Определение площади поперечного сечения сопла. Изменение площади
поперечного сечения сопла в зависимости от перепада давления.
36.
Отклонение потока в косом срезе сужающегося сопла.
37.
Нерасчетные режимы работы суживающихся сопел и сопел Лаваля.
38.
Понятие коэффициента потери полного давления.
39.
Одномерное изоэнтропийное течение в канале переменного сечения
(вывод уравнения движения).
40.
Влияние числа Маха на форму соплового канала (анализ уравнения
движения).
41.
Критические параметры потока. Газодинамические функции.
Введение
1.
Роль и место газовой динамике в судовой энергетике
Основные понятия газовой динамики
2.
3.
4.
5.
6.
7.
ние).
8.
Основные понятия газовой динамики
Классификация сил, действующих в жидкости.
Параметры потока.
Физический смысл величин: плотность, удельный вес, вязкость динамическая, вязкость статическая.
Понятие сжимаемости. Число Маха.
Понятие давления газов (полное, статическое и динамическое давлеСвязь между напряжениями и деформациями в движущемся газе.
Основные законы газовой динамики
9.
10.
11.
12.
13.
14.
Уравнение неразрывности для сжимаемого газа.
Уравнение энергии.
Уравнение количества движения.
Уравнение Бернулли
Уравнение движения идеальной жидкости
Уравнение Навье-Стокса
Основы физического моделирования
42.
43.
44.
Одномерные течения газа
15.
Понятие трехмерного, двумерного и одномерного течения.
16.
Плоские установившиеся движения идеальной сжимаемой жидкости.
17.
Понятие линии тока и функции тока.
18.
Уравнение линии тока.
19.
Отличие линии тока от траектории движения частицы газа.
20.
Понятие трубки тока и элементарной струйки газа.
21.
Понятие поверхности тока.
22.
Понятие нулевой линии.
23.
Понятие вихревой линии. Уравнение вихревой линии.
24.
Вихревая трубка тока. Уравнение вихревой трубки (теорема
Гельмгольца).
25.
Свойства вихревых трубок.
26.
Формулировка теоремы Стокса.
45.
46.
47.
Основные понятия гидродинамического подобия. Задачи моделирования
и подобия. Размерные и безразмерные величины.
Критерии подобия в газовой динамике.
Прямые скачки уплотнения. Образование волн сжатия и разряжения.
Схема распространения ударной волны.
Изменения состояния газа в скачке уплотнения. Относительная скорость
газа в скачке уплотнения.
Косые скачки уплотнения. Слабые возмущения в косом скачке
уплотнения.
Конус Маха. Предельная скорость движения газа. Число Маха.
Пограничный слой
48.
Понятие пограничного слоя. Условия его образования.
49.
Понятие ламинарного и турбулентного пограничного слоев. Изменение
скорости газа в пограничном слое.
50.
Образование пограничного слоя при обтекании потоком кругового тела.
59
Библиографический список
51.
Понятия толщины пограничного слоя, толщины вытеснения и толщины
потери импульса.
Технические приложения газовой динамики
52.
53.
54.
55.
56.
57.
58.
59.
60.
61.
1. Абрамович Г.Н. Прикладная газовая динамика. – М.: Наука,
1976. – 888 с.
Течение газа по трубам.
Газовые эжекторы.
Течение газа с подводом и отводом тепла.
Течение газа в лабиринтных уплотнениях.
Основы обтекания газом решеток профилей лопастных машин (уравнения Эйлера и теорема Жуковского).
Расчет термодинамических параметров движущегося газа.
Применение основных законов газовой динамики к расчету
кинематических характеристик потока.
Расчет сужающихся сопел.
Расчет сверхзвуковых сопел.
Определение потерь энергии в трубах.
2. Альбом течений жидкости и газа /Сост. М. Ван-Дайк. – М.: Мир,
1986. – 184 с.
3. Белоцерковский О.М. Численное моделирование в механике
сплошных сред. – М.: наука, 1984. – 520 с.
4. Бронштейн И.Н., Семендяев К.А. Справочник по математике для
инженеров и учащихся втузов. – М.: Наука. Главная редакция физикоматематической литературы, 1981. – 719 с.
5. Варгафтик Н.Б. Справочник по теплофизическим свойствам газов и жидкостей. – М.: Наука, 1972. – 721 с.
6. Виноградов Б.С. Прикладная газовая динамика. – М.: 1965. – 328
с.
7. Дейч М.Е. Техническая газодинамика. – М.: Энергия, 1974. – 592
с.
8. Идельчик И.Е. Справочник по гидравлическим сопротивлениям.
– М.: Госэнергоиздат, 1960. – 464 с.
9. Кириллин В.А., Сычев В.В., Шейдлин А.Е. Техническая термодинамика. – М.: Наука, 1979. – 512 с.
10. Кириллов И.И. Теория турбомашин. М.: Машиностроение, 1964.
– 410 с.
11. Корн Г., Корн Т. Справочник по математике: для научных работников и инженеров. М.: Наука, Главная редакция физико-математической
литературы, 1968. – 720 с.
12. Кочин Н.Е., Кибель И.А., Розе Н.В. Теоретическая гидромеханика: часть первая. – М.: Государственное изд-во физико-математической литературы, 1963. – 584 с.
13. Курзон А.Г. Теория судовых паровых и газовых турбин. Л.: Судостроение, 1970. – 592 с.
60
14. Кухлинг Х. Справочник по физике. – М.: Мир, 1982. – 520 с.
28. Самойлович Г.С., Нитусов В.В. Сборник задач по гидроаэроме-
15. Ламб Г. Гидродинамика. Т. 1. – Москва-Ижевск: НИЦ "Регуляр-
ханике: Учебн. пособие для студентов вузов, обучающихся по специально-
ная и хаотическая динамика", 2003. – 452 с.
сти "Турбиностроение". – М.: Машиностроение, 1986. – 152 с.
16. Ландау Л.Д., Лифшиц Е.М. Теоретическая физика: Т. 6, Гидро-
29. Седов Л.И. Методы подобия и размерности в механике. - М.:
динамика. – М.: Наука, 1988. – 776 с.
Наука, 1981, 447 с.
17. Лойцянский Л.Г. Механика жидкости и газа. – М.: Наука, 1973. –
30. Седов Л.И. Механика сплошной среды. Т. 1 – СПб.: Изд-во
847 с.
"Лань", 2004. – 528 с.
18. Международная система единиц /Г.Д. Бурдун, Н.В. Калашников,
31. Седов Л.И. Механика сплошной среды. Т. 2 – СПб.: Изд-во
Л.Р. Стоцкий М.: Высшая школа. – 274 с.
"Лань", 2004. – 560 с.
19. Меркулов В.И. Гидродинамика знакомая и незнакомая. – М.:
32. Фабрикант Н.Я. Аэродинамика: часть первая. – Л.: Государ-
Наука, 1989. – 136 с.
ственное изд-во технико-теоретической литературы, 1949. – 624 с.
20. Паровые и газовые турбины: учебник для вузов /М.А. Трубилов,
33. Физический энциклопедический словарь /Гл. ред. А.М. Прохоров.
Г.В. Арсеньев, В.В. Фролов и др.; под ред. А.Г. Костюка, В.В. Фролова. – М.:
Ред. Кол. Д.М. Алексеев, А.М. Бонч-Бруевич, А.С. Боровик-Романов и др. –
Энергоатомиздат, 1985. – 352 с.
М.: Сов. энциклопедия, 1983. – 928 с.
21. Повх И.Л. Техническая гидромеханика. Л.: Машиностроение,
34. Черный Г.Г. Газовая динамика: Учебник для университетов и
1976. – 504 с.
втузов. – М.: Наука, 1988. – 424 с.
22. Политехнический словарь /Гл. ред. И.И. Артоболевский. М.: Со-
35. Яворский Б.М., Детлаф А.А. Справочник по физике для инжене-
ветская энциклопедия, 1976. – 608 с.
ров и студентов вузов. – М.: Наука, 1979. – 944 с.
23. Прандтль Л., Титьенс О. Гидро- и аэромеханика: Т. 1. Равновесие движения жидкостей без трения. М.: Технико-теоретическое издательство, 1933. – 223 с.
24. Ривкин С.Л. Термодинамические свойства газов: справочник. –
М.: Энергоатомиздат, 1987. – 288 с.
25. Ривкин С.Л., Александров А.А. Теплофизические свойства воды
и водяного пара. - М.: Энергия, 1980, 424 с.
26. Самойлович Г.С. Гидроаэромеханика. - М.: Машиностроение,
1980, 280 с.
27. Самойлович Г.С. Гидрогазодинамика: учебник для студентов вузов, обучающихся по специальности "Турбиностроение". М.: Машиностроение, 1990. – 384 с.
61
Приложение А
(справочное)
Термодинамические свойства воздуха и водяного пара
Таблица А1 Термодинамические свойства водяного пара
(μ =18,016 моль, R=461,5 Дж/(кг К)
Таблица А1 Термодинамические свойства воздуха
(μ =28,97 моль, R=287 Дж/(кг К)
t°C
TК
cp Дж/(кг
К)
cv Дж/(кг
К)
k
0
273,15
1859,7
1398,2
1,330
25
298,15
1864,4
1404,4
1,328
50
323,15
1871,4
1409,9
1,327
75
348,15
1880
1418,5
1,325
223,15
cp Дж/(кг
К)
1002
cv Дж/(кг
К)
715,0
1,4014
-25
248,15
1002,3
715,3
1,4012
0
273,15
1002,8
715,8
1,4009
100
373,15
1890
1428,6
1,323
25
298,15
1003,8
716,8
1,4004
125
398,15
1901,2
1439,7
1,321
50
323,15
1005,3
718,3
1,3996
75
348,15
1007,3
720,3
1,3984
150
423,15
1913,4
1451,9
1,318
100
373,15
1009,8
722,8
1,3971
175
448,15
1926,3
1464,8
1,315
125
398,15
1012,8
725,9
1,3952
200
473,15
1939,9
1478,4
1,312
150
423,15
1016,3
729,3
1,3935
175
448,15
1020,2
733,2
1,3914
250
523,15
1968,8
1507,3
1,306
200
473,15
1024,4
737,4
1,3892
300
573,15
1999,4
1538,0
1,300
250
523,15
1033,9
746,9
1,3843
350
623,15
2031,5
1570,0
1,294
300
573,15
1044,5
757,5
1,3789
350
623,15
1055,9
770,0
1,3713
400
673,15
2064,6
1603,1
1,288
400
673,15
1067,8
780,8
1,3676
450
723,15
2098,4
1636,9
1,282
450
723,15
1079,8
792,8
1,362
500
773,15
2132,9
1671,4
1,276
500
773,15
1091,8
804,8
1,3566
550
823,15
1103,6
816,6
1,3515
550
823,15
2167,7
1706,3
1,270
600
873,15
1115
828,0
1,3466
600
873,15
2203
1741,5
1,265
650
923,15
1125,8
838,8
1,3422
650
923,15
2238,3
1776,9
1,260
700
973,15
1136,1
849,1
1,338
750
1023,15
1145,7
858,7
1,3342
700
973,15
2273,8
1812,3
1,255
800
1073,15
1154,6
867,6
1,3308
750
1023,15
2309,1
1847,6
1,250
850
1123,15
1162,9
876,0
1,3275
800
1073,15
2344,1
1882,7
1,245
900
1173,15
1170,7
883,7
1,3248
950
1223,15
1177,9
890,9
1,3221
850
1123,15
2378,8
1917,4
1,241
1000
1273,15
1184,6
897,6
1,3197
900
1173,15
2413
1951,5
1,236
t°C
TК
-50
k
62
t°C
TК
cp Дж/(кг
К)
cv Дж/(кг
К)
k
950
1223,15
2446,6
1985,1
1,232
1000
1273,15
2479,3
2017,8
1,229
1050
1323,15
2511,2
2049,7
1,225
1100
1373,15
2542
2080,5
1,222
1150
1423,15
2571,8
2110,3
1,219
1200
1473,15
2600,4
2138,9
1,216
1250
1523,15
2627,9
2166,4
1,213
1300
1573,15
2654,1
2192,7
1,210
1350
1623,15
2679,3
2217,8
1,208
1400
1673,15
2703,4
2241,9
1,206
1450
1723,15
2726,7
2265,2
1,204
1500
1773,15
2749,4
2288,0
1,202
Приложение Б
(справочное)
Таблица Б1 Динамическая вязкость газов (при 20°С и 101,3 кПа)
Название газа
63
Динамическая вязкость
(мПа с)
Азот
0,0175
Аммиак
0,00995
Водород
0,0088
Воздух
0,0182
Гелий
0,0196
Двуокись углерода
0,0147
Кислород
0,0202
Метан
0,0108
Окись углерода
0,0177
Приложение В
Оглавление
(справочное)
Условные обозначения . . . . . . . . . . . . . . . . . . . . . . . . . . . . . . . . . . . . . . . . . . .
3
Таблица В1 Газовая постоянная
Введение . . . . . . . . . . . . . . . . . . . . . . . . . . . . . . . . . . . . . . . . . . . . . . . . . . . . . . .
5
Название газа
Азот
Аммиак
Газовая постоянная
(Дж/(кг К)
1. Основные гипотезы механики сплошной среды . . . . . . . . . . . . . . . . . .
8
2. Классификация сил, действующих в газе. . . . . . . . . . . . . . . . . . . . . . . . .
12
297
3. Параметры состояния газа . . . . . . . . . . . . . . . . . . . . . . . . . . . . . . . . . . . .
15
488
Водород
4125
Воздух
287
Гелий
2078
Двуокись углерода
189
Кислород
260
Метан
519
Окись углерода
297
4. Физический смысл некоторых величин . . . . . . . . . . . . . . . . . . . . . . . . .
16
4.1. Давление . . . . . . . . . . . . . . . . . . . . . . . . . . . . . . . . . . . . . . . . . . . . . . . . . .
16
4.2. Температура. . . . . . . . . . . . . . . . . . . . . . . . . . . . . . . . . . . . . . . . . . . . . . . .
18
4.3. Плотность . . . . . . . . . . . . . . . . . . . . . . . . . . . . . . . . . . . . . . . . . . . . . . . . .
20
4.4. Удельный вес . . . . . . . . . . . . . . . . . . . . . . . . . . . . . . . . . . . . . . . . . . . . . .
22
4.5. Удельный объем . . . . . . . . . . . . . . . . . . . . . . . . . . . . . . . . . . . . . . . . . . .
22
4.6. Вязкость . . . . . . . . . . . . . . . . . . . . . . . . . . . . . . . . . . . . . . . . . . . . . . . . . .
23
4.6.1. Динамическая вязкость . . . . . . . . . . . . . . . . . . . . . . . . . . . . . . . . .
27
4.6.2. Кинематическая вязкость . . . . . . . . . . . . . . . . . . . . . . . . . . . . . . . .
28
4.7. Сжимаемость . . . . . . . . . . . . . . . . . . . . . . . . . . . . . . . . . . . . . . . . . . . . . . .
29
4.8. Число Маха . . . . . . . . . . . . . . . . . . . . . . . . . . . . . . . . . . . . . . . . . . . . . . .
30
4.9. Энтальпия . . . . . . . . . . . . . . . . . . . . . . . . . . . . . . . . . . . . . . . . . . . . . . . . .
32
4.10. Энтропия . . . . . . . . . . . . . . . . . . . . . . . . . . . . . . . . . . . . . . . . . . . . . . . .
33
64
Вопросы для самопроверки . . . . . . . . . . . . . . . . . . . . . . . . . . . . . . . . . . . . . . .
35
Вопросы для самопроверки . . . . . . . . . . . . . . . . . . . . . . . . . . . . . . . . . . . . . .
100
5. Закон сохранения массы . . . . . . . . . . . . . . . . . . . . . . . . . . . . . . . . . . . . . . .
36
Библиографический список . . . . . . . . . . . . . . . . . . . . . . . . . . . . . . . . . . . . . .
102
6. Закон сохранения энергии . . . . . . . . . . . . . . . . . . . . . . . . . . . . . . . . . . . . .
40
Приложение А Термодинамические свойства воздуха и водяного пара . . . .
105
7. Уравнение количества движения . . . . . . . . . . . . . . . . . . . . . . . . . . . . . . .
52
Приложение Б Динамическая вязкость газов . . . . . . . . . . . . . . . . . . . . . . . . .
107
8. Уравнение моментов количества движения . . . . . . . . . . . . . . . . . . . . . .
58
Приложение В Газовая постоянная . . . . . . . . . . . . . . . . . . . . . . . . . . . . . . . .
108
9. Уравнение Бернулли . . . . . . . . . . . . . . . . . . . . . . . . . . . . . . . . . . . . . . . . .
64
9.1. Трубка Прандтля - Пито . . . . . . . . . . . . . . . . . . . . . . . . . . . . . . . . . . . . .
70
9.2. Трубка Вентури . . . . . . . . . . . . . . . . . . . . . . . . . . . . . . . . . . . . . . . . . . . .
71
9.3. Шарик висит в воздухе . . . . . . . . . . . . . . . . . . . . . . . . . . . . . . . . . . . . . .
74
9.4. Эффект Магнуса . . . . . . . . . . . . . . . . . . . . . . . . . . . . . . . . . . . . . . . . . . . .
75
Приложение Г Задачи для самостоятельного решения . . . . . . . . . . . . . . . . .
109
10. Теоретические параметры газа при его движении . . . . . . . . . . . . . . . .
79
11. Примеры решения задач . . . . . . . . . . . . . . . . . . . . . . . . . . . . . . . . . . . . . .
84
11.1. Определение изменения энтропии . . . . . . . . . . . . . . . . . . . . . . . . . . .
84
11.2. Определение скорости газа с помощью закона сохранения массы .
85
11.3. Определение температуры с помощью закона сохранения энергии
86
11.4. Определение необходимого противодавления
с помощью уравнения количества движения . . . . . . . . . . . . . . . .
. . 88
11.5. Течение газа в канале . . . . . . . . . . . . . . . . . . . . . . . . . . . . . . . . . . . . . .
89
11.6. Воздушный эжектор . . . . . . . . . . . . . . . . . . . . . . . . . . . . . . . . . . . . . . .
92
65
Download